terça-feira, 31 de dezembro de 2013

O senhor pode me dizer o que é discalculia? E se é possível uma pessoa ter?‎ aninha

http://pt.wikipedia.org/wiki/Discalculia

Na sequência definida por "N(i) = D(i,1) + N(i-1) + i - 1", o que acontece quando i=1, já que eu preciso do N(i-1) que nesse caso é o N(0)?‎ ბრუნო

Faltou a definição adicional de que N(0)=0 e que i>=0.

Mas o sr não acha que em um povo pouco civilizado e inculto, a tirania é o único método para o desenvolvimento e progresso? O Brasil por exemplo, se desenvolveu muito mais durante os anos da Ditadura, do que durante as democracias.‎ Radioactive

De forma nenhuma! Claro que não. Ditaduras e regimes tirânicos são abomináveis, mesmo que o ditador seja esclarecido e bem intencionado. Porque elas permitem a proliferação de injustiças, de opressões por parte dos funcionários e militares. Não dão ao povo o direito de protestar, de exigir justiça. São terríveis. A cultura e a civilização têm que ser alcançadas democraticamente, mesmo que demore mais. Os fins jamais justificam os meios. Além disso não é verdade que o Brasil se desenvolveu mais durante a ditadura militar. Na do Getúlio o progresso foi maior. E sob Juscelino, maior ainda. Os governos posteriores, cada qual, promoveram outras conquistas, inclusive o do Collor. Também houve progresso econômico no regime militar, mas se não fosse ele, se o Brasil continuasse democrático, haveria também, sem as perversidades da ditadura.

O que diferencia crianças de adultos? fiz essa pergunta ao meu pai, e ele falou atitudes. concorda? também queria perguntar ao senhor. Qual a base de um bom relacionamento? o senhor ja namoro quantas vezes. E porque esses namoro acabaram?por enquanto é só‎ HiggorNogueira

O que diferencia, além da estatura, é o nível de desenvolvimento das conexões cerebrais e o volume de experiência armazenado na memória. A própria capacidade de raciocínio abstrato, o controle emocional e várias outras funções psíquicas vão se desenvolvendo aos poucos, só estando tudo pronto lá pelos 23 anos, quando se pode dizer que se atingiu a maturidade, orgânica e psíquica. Daí para frente vai havendo um declínio, com um certo platô até lá pelos 40 anos, mas sempre descendo e cada vez com mais declividade. O importante para um bom relacionamento com as crianças, os adolescentes e os jovens é saber, para cada idade, o que se pode esperar deles, em função da sua maturação neurológica e agir de acordo, não tendo expectativas impossíveis para eles e nem aquém do que sejam capazes. É preciso levá-los a sério e atribuir-lhes responsabilidades. Nunca, especialmente com crianças, considerar que o que eles dão valor seja bobeira de criança, porque não é. Faz parte do processo de crescimento, por exemplo, brincar. E brincadeira, para a criança, é uma coisa séria. O que não significa que não seja alegre. Recomendo o livro "O Cérebro em Transformação" de Suzana Herculano-Houzel (Objetiva).

Você acredita na possibilidade de que nosso dna pode ter sido modificado por seres extraterrestre milhares de anos atrás ?‎ Wilian

Não. Acho que somos produto da evolução natural dos primatas terrestres.

Caro E. Von Rückert, o sr enxerga Josef Stálin como um grande tirano ou um grande líder?‎ Radioactive

Um grande tirano. Um déspota. Nada justifica sua conduta. Tão facínora quanto Hitler, Mao Tse-Tung, e outros do tipo.

tem como o senhor explicar por que é impossível acontecer um dilúvio na terra?‎ alexandre

Porque não há água nos oceanos suficiente para tal. A água total de Terra é constante. A que cai das chuvas vem do oceano e volta para ele. Para elevar o nível dos oceanos até cobrir a mais alta montanha da Terra se gastaria quase o triplo da água existente nos oceanos. Supondo S a superfície da Terra e H a altura da montanha mais alta, o volume necessário de água seria SH (desprezando os termos quadráticos e cúbicos da expressão). Se os oceanos ocupam 3/4 da superfície e têm uma profundidade média de H/2, o volume de água deles é (3/8)SH, o que significa que o aumento corresponderia a 8/3 da água de todos os oceanos. Além disso, para fazer isso em 40 dias, seria preciso chover 8000 mm/h, quando a chuva mais forte que já se viu não passou de 20 mm/h.

O que acha do preconceito que existe com pessoas que curtem rock e andam na rua vestido de preto?‎ Douglas Berna

Não me consta que existe esse preconceito. Se houver, por parte de alguém, trata-se de uma pessoa completamente ignorante, que acha que as pessoas não podem ser diferentes do comum das outras. Há gente assim. Mas isso, realmente, é ignorância da grossa. Cada qual tem todo o direito de ter um modo de ser próprio, que pode ser totalmente individualizado ou compartilhado com algum grupo. O que não pode é o modo de ser ser prejudicial ou maléfico aos outros. Se não atrapalhar nada, não tem o menor problema. Mesmo que não atrapalhe, há alguns casos que podem ser vetados, apesar de que eu não vejo problema, como andar nu na rua. Já praticar o coito em público, ou mesmo defecar e urinar, como os cachorros fazem, não acho que seja conveniente para os humanos.

O senhor acredita em viajar no tempo, mas apenas para o passado ou para o futuro também?

sso não é uma questão de acreditar, como a maioria das questões, e sim de se saber. Pelo que se sabe, viagens ao futuro a ritmos mais céleres do que o que sempre estamos indo (uma hora por hora) é possível e eu até já exemplifiquei numericamente respostas atrás. Para o passado é que não se consegue ir.

Não teria como deixar a definição mais "enxuta"? Exemplo: F(n-1)+ F(n-2) caso N>1 é a definição formal para a sequência de Fibonacci. Seria possível fazer uma definição tão simples para aquela outra sequência? Quantas leis de formação uma sequência pode vir a ter? Pode ser que não tenha nenhuma?‎ Victor

Talvez se consiga. É o caso de tentar. Claro que se pode ter mais de uma expressão que dê o mesmo resultado. Umas mais simples outras mais complexas. Não existe limite de número de modos de se apresentar uma expressão dessas. Há casos em que a lei de formação não consegue ser expressa por uma equação, mas apenas por uma expressão verbal.

Como definir formalmente a sequência: 1, 2 , 4, 7, 11, 16..?Eu já entendi que o próximo número da sequência é o anterior mais o número da ordem do próprio anterior, mas não consigo ver uma maneira fácil de definir formalmente a sequência. Existe alguma dica para definir com facilidade as sequências?‎ Victor

N(i) = D(i,1) + N(i-1) + i - 1, onde N(i) é o iésimo termo, D(i,j) é o "delta de Kronecker" que vale 1, para i = j e vale 0, para i <> j. i é o número de ordem do termo. Esse tipo de expressão é obtida por tentativa, após inspeção da sequência. Não há um método geral de obtenção. Tem que ter treino para isso, além de intuição. Não existe dica para se achar, com facilidade, essas expressões gerais. O recurso é achar com dificuldade mesmo.

Olá Ernesto! Não sei se já lhe perguntaram, mas tenho uma curiosidade. Quando os ateus morrem vocês são enterrados em caixão, com orações e tal ou não? Creio eu que vocês não pagam dizímo então como é feito tudo isso? Agradeço se me responder e tirar essa dúvida, obrigada!‎ Andressa

Sepultamento ou cremação não são ocorrências religiosas. São civis. Os cemitérios são públicos, para que sejam enterradas pessoas de qualquer religião ou de nenhuma. Os velórios são feitos em lugares públicos e não religiosos. As pessoas de cada religião é que encomendam serviços religiosos das funerárias. Mas elas atendem tanto a cristãos quanto a judeus, muçulmanos, budistas, hinduístas ou ateus. No caso de ateus, simplesmente acontece o velório como uma ocasião para os amigos consolarem os familiares e expressarem os seus sentimentos de pesar. Não há necessidade de haver nenhuma liturgia religiosa, e da presença de nenhum símbolo religioso de nenhuma religião. Como também não se precisa fazer nenhuma oração religiosa. O que se pode é haver discurso, mas de conteúdo totalmente laico.

Existe varios relatos q realmente existiu a ''Arca de Noé'' como vc explicaria?‎ Lucas Gomes

Ter existido uma arca não significa que ela tenha sido construída pelo personagem bíblico Noé, em atendimento a instruções divinas e nem que tenha sido usada para abrigar um casal de cada espécie animal da Terra para que não perecessem em um dilúvio que tenha submergido todos os continentes. Essas duas últimas coisas são impossibilidades totais. E, para mim, a primeira também (as instruções divinas).

O senhor concorda que a ditdura foi a grande culpada pela decadência do ensino público atual ?‎ Mandy Bunny Z™

Sim e não. A ditadura fez um bom serviço estendendo a educação a toda a população com a abolição do "Admissão ao Ginásio" e transformando os cursos primário e ginasial no Ensino Fundamental continuado, da primeira à oitava série (hoje nono ano). Mas, com isso, teve que abrir muitas vagas (e tinha mesmo), o que acarretou aumento de despesa. Para contornar, achatou o salário dos professores, restringindo os aumentos. Então o magistério deixou de ser uma opção atrativa para as melhores cabeças, ficando relegado, fora exceções, para quem não tinha condições para fazer outros cursos mais bem remunerados. Com professores menos capazes, o nível de exigência caiu, fazendo com que os anos finais do Ensino Fundamental (antigo ginasial) ficasse muito mais frouxos, especialmente nas escolas públicas. Então o Ensino Médio também afrouxou, porque a maior parte dos que vinham do fundamental não conseguiam acompanhar o nível exigido e as reprovações se avolumaram. Para evitá-las, optou-se pelo afrouxamento das exigências. Isso é que foi devastador. Que todo mundo possa fazer o Ensino Médio é algo de clareza cristalina. Que este tenha que ser facilitado para que todos consigam é um despropósito. O que se tem é que dar condições para que todos o façam de forma difícil mesmo. Facilitar é um suicídio nacional da competência do país. Retornar às condições anteriores de qualidade, sem elitizar o ensino é o grande desafio, que, para mim, vai levar uns duzentos anos para se conseguir. Para começar seria preciso pagar muito bem aos professores, de modo que as melhores cabeças PREFERISSEM ser professores do que médicos ou advogados, PORQUE GANHA MAIS. Ou seja um professor primário, com dedicação exclusiva, teria que ganhar, como piso salarial, pelo menos cinco mil reais. E todos teriam que fazer mestrado e doutorado e ir melhorando de salário até o fim da carreira de professor universitário titular, ganhando uns trinta mil reais ou mais. Mas o concurso para entrar teria que ser duro. Precisaria criar uma nova carreira, a que os anteriores poderiam entrar por concurso e colocar a anterior em extinção, até que todos se aposentem, se não forem promovidos para a nova. A velha continuaria com o salário velho. Isso só para começar a conversa. Há muito mais a se fazer, contudo. Sugiro a leitura dos artigos do Ioschpe e do Cláudio de Moura Castro.

Por incrivel que pareca , sou física também , mas isso não me influenciou em nada em minha religião . Já que sua ciência explica tudo !. Como me explica o fato do mar ter se abrido , como diz na bíblia ?

Não se explica, porque, simplesmente, não se abriu. É só uma lenda.

Faço o 7 ano mas tipo óque fazer pra tirar nota boa em matemática.como fazer pra r uma boa nota em matemática.como se estuda ?‎ Belieber da pista;)

Matemática é complicado porque depende do conhecimento de todos os anos anteriores. Mesmo assim, a primeira providência, sem a qual não se consegue nada, nada mesmo, é aprender a GOSTAR de matemática. Só quem gosta é que vai ter o interesse de se debruçar horas em cima do estudo para entender profundamente de fato e conseguir solucionar os exercícios e problemas. Na aula, preste a máxima atenção e encha o professor de perguntas até ter entendido tudo. Não se incomode em pagar mico e nem com a opinião dos colegas. Se o professor não quiser atender suas dúvidas, reclame com a coordenação ou orientação. Não adianta pegar aula particular para treinar a resolver problemas. Tem é que APRENDER a teoria de modo que consiga fazer os problemas por sua própria conta. Quanto a gostar de matemática, tem alguns livros que ajudam:
Alex no País dos Números - Alex Bellos - Companhia das Letras
Incríveis Passatempos Matemáticos - Ian Stewart - Zahar
Almanaque das Curiosidades Matemáticas - Ian Stewart - Zahar.

Tenho 20 anos; ainda não entrei na faculdade, e pelo que estudei esse ano, só posso tentar de verdade ano que vem; enfrento problema de organizar horários, e eu tenho tempo. Sinto muita dificuldade em me organizar, falta de foco. Teria algum conselho? :x‎ Pimenta, Felipe

Só um. Decisão e força de vontade. Não há outro modo de vencer a procrastinação. Tem que resolver o que precisa fazer e ir fazendo com preguiça mesmo, insistindo, evitando os desvios, indo em frente. Aí vai se acostumando a fazer o que é difícil, o que cansa, o que complica, o que é chato e vai fazendo assim mesmo: chato, difícil, trabalhoso, cansativo. É o único jeito. Se não conseguir, desiste de fazer faculdade e se dedique a algo que não dê trabalho. O problema não é de se organizar e sim de não agir. Pode agir sem se organizar que funciona.

Você se diz ateu certo ? Podes saber o gosto de uma maça sem prova-lá ? Ou ao menos saber qual é sua situação física em seu interior ? Nunca podemos dizer algo que não provemos , sentimos ou olhamos com nossos olhos .Não estou dizendo que deves acreditar só estou te fazendo um convite a reflexão .

Mas eu já provei a sensação de crer em Deus. Durante minha infância e adolescência eu fui um católico fiel. Queria ser santo mesmo. Ia à missa e comungava todo dia. Só que eu estudava muito: religião, filosofia, cosmologia, física, biologia, evolução, neurologia, história, política. Meus estudos me levaram a concluir que a fé que eu tinha era totalmente despropositada. Por isso me tornei ateu. Se Deus existisse seria muito bom. É uma pena que não exista. Mas não existe.

o que fazer quando és péssimo em produções de textos?‎ Elaine

Ler muito. Muito mesmo. Umas duas horas por dia, anos a fio. Nos sábados, domingos e feriados, leia mais. Veja menos televisão, durma menos, entre menos na internet, namore menos e leia mais. E escrever. Uma redação por dia. Meia hora escrevendo. Mas não é dessas curtinhas de só 25 linhas não. Escreva, pelo menos, umas 100 linhas. Aí você mesmo corrige a sua redação ou pede a um professor para corrigir, prestando atenção no que errou para não errar mais. Se você insistir nisso, aprenderá a escrever em um ano.

Caro E. Von Rückert, você sente orgulho ou vergonha de ser brasileiro?‎ Radioactive

Sinto mais vergonha do que orgulho. Mas não quero abandonar o Brasil. Quero lutar para melhorar isto aqui. Tenho colaborado no aspecto educacional, tendo dado mais de 20 mil aulas de física e matemática para mais de 4 mil alunos. E, em minhas aulas, eu sempre abordo os problemas sociais, políticos, éticos, filosóficos e correlatos. Não é só conhecimento e habilidades. Também educo o caráter e a personalidade. Procuro corrigir esse modo brasileiro de "dar um jeitinho", "quebrar o galho" e todo esse tipo de comportamento indolente e licencioso que abomino. Essa fobia pela dificuldade, pelo trabalho, pela complicação, que minam o poder inventivo e desenvolvimentista. Porque o que dá certo e funciona é o que é complicado, é o que dá trabalho, é o que cansa, é o que é difícil. Temo muito por nosso futuro quando essa geração de preguiçosos e medíocres se tornar a comandante da nação. Não teremos capacidade para nada e nos tornaremos reféns dos povos mais inteligentes, mais esforçados, mais cultos, mais desenvolvidos. Porque não tivemos pertinácia e obstinação para vencer as dificuldades. Por isso é que, em minhas aulas, não facilito, pelo contrário, complico e exijo, para que os alunos aprendam a enfrentar o que é difícil. Professor que afrouxa a exigência está colaborando com a bancarrota do país. Mas eu dou condições para os alunos enfrentarem as dificuldades. Só que quem for preguiçoso não passa mesmo. Outra coisa é que eu faço treinamento para ficar mais inteligente. É uma calistenia mental. Todo professor tem que aprimorar a inteligência dos alunos. E é complicando e puxando para "fundir a cuca" que se faz isso. É com um treinamento atlético.

Quando surgiu seu interesse por matemática e física?‎ MeyMabb

Desde criança eu sempre gostei de matemática, que chamava de "boatemática". Gostava especialmente de geometria e de desenhar. Quanto à física, gostava muito de ciências. Meu pai me dava livros infantis de divulgação científica, da coleção "Enciclopédia Juvenil" da Distribuidora Record, que eram excelentes e eu adorava estudar, mesmo sem corresponder aos assuntos escolares. Livros sobre eletrônica, física atômica, astronomia, evolução, química e vários outros assuntos. Na escola, apesar de minhas melhores notas serem em Geografia e História, que quase só tirava 100%, o que eu gostava mais era de Matemática e Física, mesmo tirando menos (uns 90%). Assim eu sempre fui um deslumbrado pelo conhecimento e pelo desenvolvimento de muitas habilidades, quase sempre não relacionadas com o que se pedia na escola. Mas na escola eu me saia muito bem, mesmo estudando pouco, só pelo que aprendia durante as aulas. Em verdade, não me esforçava muito. De modo que não tenho mérito nenhum em ter sido um bom aluno, academicamente falando. Geralmente eu passava as aulas desenhando histórias em quadrinhos em meu caderno, porque o que o professor estava ensinando eu já tinha aprendido. Geralmente em algumas aulas eu pegava o livro e ia adiantando lá para frente. Depois eu podia ler outras coisas, desenhar ou escrever histórias, pois já sabia o que estava sendo dado. Os professores não reclamavam, pois eu ficava quieto e sempre respondia certo o que me perguntavam.

Então o chamado ''Universo paralelo'' é como se fosse uma ''continuidade'' do nosso? Ou é algo diferente, ainda? Inclusive, professor, o que você acha da Interpretação dos Muitos Mundos?‎ Interestelar

Universo paralelo não seria uma continuidade do nosso. Seria outro universo disjunto, isto é, totalmente incomunicável. Cada um deles seria uma realidade separada, inteiramente contida em si mesma e sem lado de fora. Só existe espaço dentro de cada Universo. Não há um espaço em que vários Universos estejam. Cada qual é completo em si mesmo, com seu tempo e seu espaço distintos. Muitos mundos seria uma situação em que, à medida que os eventos fossem evoluindo neste Universo, seriam formadas continuidades distintas, conforme as possibilidades de ocorrências, de modo que para cada conjunto de possibilidades haveria uma evolução paralela que seria um outro mundo ao longo do tempo, mas no mesmo lugar. Essa multiplicidade de mundos não é a mesma coisa que Universos paralelos, pois estes não estariam no mesmo espaço e os múltiplos mundos sim. Mas em um fluxo diferente de tempo. Isso é possível se entendermos o Universo não como o conjunto dos lugares mas como o conjunto dos eventos. Diferentes conjuntos de eventos, mesmo que tendo ocorrido nos mesmo lugares, são outros mundos. Claro que outro conjunto de eventos em outros lugares seria um outro Universo.

Já estou lendo as respostas, thanks. Outra questão: se o nosso Universo está num buraco negro, quer dizer que esse buraco negro está num outro Universo, certo? Então esse Universo surgiu de um outro buraco negro que surgiu de outro e assim por diante ou foi do Big Bang? Pode me indicar livros sobre?‎ Interestelar

Não é isso. Sermos o interior de um buraco negro pode significar duas coisas. Ou que esse buraco negro seja a totalidade do Universo, no caso dele ser finito, e não haver mais nada além disso ou esse buraco negro não ser todo o Universo e haver um resto de Universo fora dele (até maior ainda, se for finito e, certamente, maior ainda, se for infinito). O que significa é que esta porção de Universo em que estamos não tem como se comunicar com o resto do Universo, se houver. Mas o resto pode mandar conteúdo material e radiante para dentro. Note que os cálculos mostraram que o Buraco Negro em que estamos é maior do que o Universo Observável, por enquanto. De modo que não temos acesso ao limite interior desse buraco negro. Mas pode haver mais Universo, além dele. Só que não é outro Universo. É esse mesmo. Não há outro Universo.

O sr. Acha que o capitalismo está permanentemente consolidado, ou é só uma fase assim como foi o feudalismo? Acha que algum dia surgirá uma utopia que disputará com o capitalismo ou até mesmo substituí-lo?‎ Radioactive

Não acho que o capitalismo está consolidado coisa nenhuma. A humanidade tem 200 mil anos e ainda vai existir por vários milhões de anos. Muita coisa vai mudar. Mas, para mim, o capitalismo não resiste nem mais uns cinco mil anos. Vai acabar antes disso e a humanidade se tornará anarco-comunista. Vejo que essa é a tendência e, se se fizer um esforço, isso pode acontecer em mil ou dois mil anos só.

Sobre o espiritismo, já ouviu a frase "Não é preciso ver pra crer e sim crer pra ver"?‎ Jéssica Kirsch ✡

Essa é uma frase inteiramente sem sentido. Se algo existe ele se manifesta à percepção (não necessariamente à visão) creia-se ou não. E se algo não existe, não se manifestará, mesmo que se creia. A crença não faz diferença para a realidade de nada. O que pode acontecer é que, crendo, a pessoa tenha a ilusão de perceber algo que não existe como se existisse.

Caro E.Von Rückert você acha que será possível algum dia ocorrer a transição do Socialismo para o Comunismo?‎ Radioactive

Não. Para mim o caminho para o comunismo não é o socialismo e sim uma pulverização total do capitalismo de modo a transformar toda pessoa em capitalista e acabar completamente com o trabalho assalariado. Paralelamente vai se fazendo o máximo de empreendimentos comunitaristas e cooperativos, sem envolvimento de dinheiro. Também vai se compartilhando as propriedades entre grupos cada vez maiores de pessoas. Até que o dinheiro e a propriedade deixem de existir por falta de necessidade, sem precisar decretar a sua abolição. Do mesmo modo, se todo mundo for fazendo o que geralmente os governos fazem em mutirões comunitários, os governos poderão deixar de existir por falta de necessidade. Chegará um dia em que ninguém vai querer ser político. Então a política vai acabar, os governos vão acabar, a propriedade vai acabar, o dinheiro vai acabar, a família mononuclear vai acabar, as fronteiras vão acabar, os estados vão acabar. Espontaneamente. E numa sociedade super avançada e sofisticada cultural e tecnologicamente. Sem pobreza, sem ignorância, sem doença. O socialismo requer um governo forte para planejar e controlar a economia. Isso é péssimo.

O senhor acredita que há pessoas realmente pessoas que que possui "dons" ( ler mentes, ver auras,ver pessoas mortas) ? Se não, por que eles tem gente que diz ter esses dons ?‎ Julia Testoni

Claro que não. Isso, ou é um vergonhoso embuste, isto é, uma tapeação sem piedade, ou é algo que a pessoa tem a impressão de ver, como se fosse uma alucinação, e acredita que seja uma leitura de mentes ou outros casos do tipo. Videntes que cobram consulta, são estelionatárias. É um crime tapear o povo crédulo.

O que você pensa sobre Chico Xavier?‎ Sérgio

  Sérgio https://www.google.com.br/search?q=inurl%3Awolfedler.blogspot.com+Chico+Xavier&oq=inurl%3Awolfedler.blogspot.com+Chico+Xavier&aqs=chrome..69i57j69i58.58428j0j4&sourceid=chrome&espv=210&es_sm=122&ie=UTF-8

Tenho um professor de filosofia muito parecido com o senhor, o sonho dele é ter todo o conhecimento do mundo(ele sabe e afirma que isso é impossível) gostaria de saber se esse também é o seu sonho ?‎ Victória Miotto

Quando eu acreditava em Deus e que tinha uma alma imortal eu ficava doido para morrer para poder ir para o céu e ter papos infindáveis com Deus para aprender tudo sobre todas as coisas. Agora sei que isso não vai acontecer e que é impossível saber de tudo. Mesmo assim sempre tenho uma curiosidade infindável e quero saber o máximo sobre tudo. Não busco utilidade nenhuma nesse conhecimento. É pelo simples prazer de saber, que, para mim, é um dos maiores da vida, depois de amar e de pensar. Mais do que comer ou dormir.

Mestre, o senhor acredita em que muita gente chama de "destino" ?‎ Sawyer

Claro que não. Isso já disse várias vezes. Simplesmente não existe.
http://spring.me/wolfedler/q/417111657852979448
http://wolfedler.blogspot.com.br/2012/08/o-destino-de-cada-um-esta-de-acordo-com.html
http://wolfedler.blogspot.com.br/2011/03/porque-acreditamos-tanto-em-destino.html

Lindo, não acha? http://www.youtube.com/watch?v=5g2sY1pUZdc‎ Guilherme Sousa

Sem dúvida. Tenho dois long-plays dele, bem como de outros da época, como Sylvio Caldas, Orlando Silva, Osny Silva, Nelson Gonçalves, Jorge Goulart, Cauby Peixoto, Francisco Alves. Bem como das divas, Dalva de Oliveira, Emilinha Borba, Ângela Maria, Marlene, Carmen Miranda, Ademilde Fonseca, Hebe Camargo, Araci de Almeida e tantos outros e outras daqueles bons tempos da Música Popular Brasileira. Eu mesmo gosto de cantar essas músicas e até que canto bem.

Professor, o senhor pode dá dicas de como estudar melhor pro PAS?‎ Rafaela Sales

Não há como estudar melhor para isto ou aquilo. Há como se estudar para "aprender" e, como isso, passar em qualquer exame. Para se aprender, antes de tudo é preciso estar querendo saber aquilo, não como uma obrigação, mas porque quer saber mesmo, porque gosta de saber. Senão não aprende. Não adianta. Por obrigação não se aprende nada. Melhor desistir. Ou se quer porque quer saber mesmo ou não se aprende. Uma vez querendo, há que se mergulhar no assunto de cabeça, revirar tudo de cabeça para baixo e, então, escrever uma apostila para que os outros aprendam com o que você está ensinando. É ensinando que se aprende. Mas não é da noite para o dia. É ao longo dos anos. De repente, querer aprender tudo o que não aprendeu em sete anos em um ano só, é quase impossível. Mas vale tentar, se se dedicar integralmente e esquecer do resto da vida. Horário integral mesmo, sem refresco, sem baladas, sem namoro, sem sábados, sem domingos, sem televisão, sem internet. O segredo é preparar aula do que está estudando. Montar esquema para explicar para os outros. Descobrir estratégias de argumentação para convencer. É assim que se aprende.

Além de música erudita, que tipo de música você ouve? O que é de popular que você ouve?‎ Kirk Lazarus

Ouço o que chamam de "Música Popular Brasileira", que inclui a Bossa Nova, a Jovem Guarda, a Tropicália e outros movimentos mais recentes de música de qualidade. Ouço samba da velha guarda, tango, jazz, blues, "Easy Music", do tipo Frank Sinatra e similares, bem como música internacional equivalente (francesa, italiana, espanhola, mexicana), boleros, "New Age" (Ênia, Vangelis e similares). Não gosto de Axé, Funk, Sertanejo Universitário (mas aprecio a caipira de raiz), Forró (mas gosto de algum Baião). Quanto ao Rock, depende. Gosto de Beatles e não gosto de Rolling Stones. Gosto de Elvis Presley. Gosto de algumas bandas de Heavy Metal também.

Vai fazer o ENEM?

Não, mas ando pensando em fazer, para checar meu nível de conhecimento. Talvez faça o ano que vem. Acho que todos os professores deveriam fazê-lo todo ano. É importante que professores, de qualquer matéria, estejam sempre atualizados com os conhecimentos do nível médio de todas as outras. Isso, para mim, é essencial para que possam, de fato, serem professores interdisciplinares. Claro que, em nível superior, cada um entende da sua. Mas em nível médio todo mundo tem que entender de tudo, qualquer que seja a área, já que é um conhecimento elementar. Nem é básico, pois básico é o conhecimento dos anos iniciais das graduações, especializado é o dos anos finais e avançado da pós-graduação. A educação básica (fundamental e médio), em todas as suas áreas, tem que ser de domínio geral para quem quer que tenha curso superior.

O que o senhor acha sobre ufologia? Acha que vale a pena se especializar nisso?‎ Yulia Kuznetsov ♡

De forma nenhuma. Mesmo que seja uma pesquisa séria, há tanta gente esotérica envolvida com isso que só de se estar no meio do assunto já se pode pensar que não se seja sério. Se você gosta mesmo, vá em frente. Mas munido de muito ceticismo e só apresentando respostas muito bem alicerçadas e não baseadas em meras suposições e palpites. E, principalmente, muita coragem para desmascarar os embusteiros. Isso é importante porque essa é uma área em que a falcatrua campeia solta.

Mestre, qual sua opinião sobre o livro "Eram os Deuses Astronautas ?" ?‎ Sawyer

Totalmente equivocado em suas propostas de explicação dos fatos arqueológicos por meio da intervenção de extraterrestres. Tudo fantasia sem nenhuma base.

Professor Ernesto, a quem cabe necessariamente o ônus da prova?‎ Q.

A quem proponha algo que não seja evidente. Tudo o que não for evidente tem que ser provado. Ou então a quem proponha uma alternativa a algo que esteja provado, mesmo não sendo evidente.

Sou estudante de história e apaixonado pela disciplina.Hoje tive minha primeira experiência em sala e ela foi HORROROSA.Fazia tempo que não sentia tão decepcionado.Havia 2 alunos prestando atenção em mim numa sala de 40. Na hora, quis até chorar.É normal isso pra quem inicia?Qual conselho vc me dá?‎ Kirk Lazarus

Sim. É normal. A moçada hoje está super desligada de aula. Só há uma solução. Fazer de sua aula um Show. Um espetáculo cenográfico muito bem planejado e ensaiado. Motivar a turma com o que eles consideram relevante, já que não se ligam na história em si mesma. Então tem que chegar nela por outras vias. Quem sabe pela música popular, pelas danças, pelos costumes da vida privada, por algum filme histórico da época abordada, por uma ligação com algo relevante da atualidade. Não sei, tem que dar tratos à bola. E, principalmente, não dar aula expositiva. Jamais. Fazer os próprios alunos descobrirem por si mesmo o que se pretende. Levá-los a escrever uma novela em grupo e representar. Tocar a música da época. Não sei. Invente. Agite. Meta a mão na massa com eles. Transforme-os em historiadores. Em contestadores. Mande-os descobrir todas as vilanias da época. As falcatruas. Critique. Diga coisas erradas e peça para eles contestarem. Faça debates com equipes a favor e contra algum fato. Assim você agita e faz a turma se interessar.

segunda-feira, 30 de dezembro de 2013

Caro E. Von Rückert, se o Brasil é um país laico, então porque existem tantos feriados religiosos? Por que o nosso ''símbolo'' é a estátua do Cristo Redentor no RJ? Até nas notas de dinheiro tem ''Deus seja louvado''. Não seria uma falsa Laicidade?‎ Radioactive

Sim. Não acho que deveria haver feriado religioso nenhum, nem crucifixos nos plenários legislativos, nas salas de juri, nos gabinetes dos executivos públicos e nem a frase "Deus Seja Louvado" nas cédulas. Quanto à estátua do Cristo Redentor, não acho que deva ser derrubada, pois já se tornou um símbolo do Rio e se trata de uma obra de arte. Do mesmo modo que as catedrais, as mesquitas, os pagodes, as estátuas de Buddha e outros que representam a cultura do seu povo.

Professor, hoje li uma matéria onde físicos encontraram evidências que a nossa realidade pode ser apenas uma simulação virtual, o que acha sobre isso?‎ Jackie

Para mim é uma concepção totalmente fantasiosa e sem fundamento nenhum. É que eles concebem que seria possível e já passam a considerar que, de fato, o seja. Isso também acontece com outras conjecturas, como a dos Universos Múltiplos, dos "túneis de minhoca", das supercordas e outras.

Professor, por favor faça um breve resumo sobre a maçonaria. Estou em uma discussão com pessoas que ainda acreditam que se trata de uma ordem ruim ou até mesmo uma religião. Vindo de mim não vão se convencer do que realmente se trata.‎ Bruno Crispim

Não gosto nada da Maçonaria, mas não tenho conhecimentos grandes sobre ela. Sei que não é uma religião e nem é "do mal", nas não gosto de que seja hermética e esotérica, isto é, cultiva conhecimentos reservados apenas aos "iniciados". Para mim todo conhecimento tem que ser divulgado para que todos tirem proveito. Além disso, ele promove uma ajuda mútua apenas entre seus seguidores. Acho que a solidariedade e o auxílio mútuo têm que ser estendidos a todo mundo. Outra coisa que não gosto é o seu caráter ritualístico, com toda uma liturgia, isso sim, parecido como uma religião. Jamais me filiaria à Maçonaria. Além de que minhas concepções anarquistas não apreciam o tipo de hierarquia que existe lá. Outro aspecto que discordo dela é sua concepção teísta do Universo e sua não aceitação do ateísmo.

Uma vez você me disse que químicos eram mais pé no chão. Mas sempre que descobriam que as coisas não eram como pensavam, buscavam descobrir mais sobre o mundo de acordo com as novas concepções, deixando todo o anterior para trás. Já os físicos, não param até descobrirem o que quer ou morrer. Você n‎ André Rodrigues

De fato, em geral os químicos vêem o lado prático e utilitário de suas descobertas. Já os físicos não. Eles buscam compreender o funcionamento do Universo sem se importar que utilidade tenha isso. Em geral eles propõem um modelo e esgotam as possibilidades de testá-lo, não desistindo enquanto algum fato empírico mostre que ele seja inviável. É o que está acontecendo com a Teoria das Supercordas. Ainda não foi comprovada, mas não se mostrou que seja inviável também. Químicos já teriam desistido dela. Meu espírito é mais de físico do que de químico. Não me importo com a utilidade do conhecimento. Acho que ele deve ser perseguido como um bem em si mesmo. Apesar de achar que a teoria das cordas não corresponda à realidade, penso que deva ser investigada até que se mostre, de fato, inviável, ou que se mostre correta.

Se a mente é gerada pelo substrato material (cérebro) e se reduz a ele, pode o ser humano um dia criar uma inteligência artificial pensante? Seremos nós capazes, então, de produzir um cérebro artificial?‎ Carlos Matos

Certamente que sim. É só uma questão de se desenvolver a tecnologia necessária. Mas isso ainda está muito longe de ser conseguido. Não é uma questão de algumas décadas e sim de séculos de progresso tecnológico. A complexidade é extrema. Centenas de vezes mais complicado do que o mais complicado sistema que já se construiu até hoje. Essa mente artificial não apenas poderá raciocinar, como também sentir emoções, ter sentimentos e intuições. E pode, até, ser mais desenvolvida do que a mente humana, sendo capaz de proezas que não conseguimos, como sintonizar ondas de rádio e FM diretamente, ter visão em outros comprimentos de onda, perceber campos elétricos e magnéticos e se orientar por eles, tendo uma espécia de "bússola" interna. O interessante também será desenvolver um processo de transferência de registros de memória de um cérebro biológico para esse. Nada disso é impossível. Apenas dificílimo.

Segundo minha professora de física, ela diz que a física é só questão de interpretação e que a parte de cálculos é só uma minoria.É usando a interpretação que eu me dou mal nas provas e nunca entendo o assunto, achas que a teoria dela esta certa?E o que eu posso fazer para melhorar nessa disciplina?‎ Çabryna

De fato, o essencial é compreender o que está acontecendo para, então, saber que fórmula usar, pois cada uma se aplica a um tipo de situação. Infelizmente muitos livros e muitos professores não se dedicam suficientemente a promover o aprendizado conceitual de Física. Para tal recomendo o livro: "Física Conceitual" de Paul Hewitt (Bookman). Você deve sempre perguntar à professora, para cada fórmula, em que situações ela se aplica e não se aplica, bem como indagar como proceder para identificar que tipo de fenômeno está acontecendo. Esse livro pode ser baixado, de graça, pela internet:
http://www.degracaemaisgostoso.org/download-fisica-conceitual-paul-g-hewitt.html

Professor, o que o senhor acha sobre Hercólubus? Tem alguma informação para compartilhar?‎ Firmino Area De Souza

Tanto Hercóbulus, quanto Nibirus e o Planeta X são ilações sem fundamento nenhum. Não existem, simplesmente. O pessoal com veleidades esotéricas gosta de considerar esse tipo de coisa, bem como Atlântida, deuses astronautas e uma série de conjecturas completamente gratuitas. Não existe nada disso. Da mesma forma que previsões do Calendário Maia, profecias de Nostradamus e similares. Se houvesse esses planetas (alguns dizem sóis) eles já teriam sido observados, inclusive, por astrônomos amadores e isso seria de conhecimento generalizado, sem possibilidade de se ocultar.

Você acha que o Amor é uma questão de Física ou Metafísica? Me parece que o verdadeiro romântico é aquele que tenta compreender o amor mas não quer sua realização, essa é a parte metafísica. Por outro lado há aqueles que vem o Amor uma espécie de força real que une as pessoas,essa é a parte física.‎ Marlon Di Gregori

Eu diria que é uma misto de Biologia e Psicologia. Ou seja, tem um componente orgânico na atração física e um componente mental, no amor mesmo. Pode haver atração sem amor e amor sem atração. O bom é haver as duas coisas. Qualquer um que reflita suficientemente sobre o significado da vida pode concluir que o amor é algo essencial à vida. Logo não se pode deixar de ser um(a) grande amante, não só de sua(seu) namorada(o) ou esposa(marido), mas de toda a humanidade, da vida, da natureza. Amor é um sentir, um pensar, um desejar, um querer e um agir, isto é, uma emoção, um sentimento, um apetite, uma volição e uma ação. Mas também é uma intelecção, refletida, consentida, explicitada e assumida. Esse complexo de fatos psíquicos caracteriza o amor em todos os planos, piedade, compaixão, solidariedade, afeição, amizade, amor platônico, erotismo. Amor filial, maternal, paternal, fraternal, conjugal, idealista. A intensidade e a sequência em que eles aparecem pode variar. O amor sempre emociona, enternece, e envolve um desejo de zelo, cuidado e proteção da coisa amada, bem como um desejo de reciprocidade. Mas o amor só se realiza quando é expresso em vontade e essa vontade em ação. Não basta sentir e desejar para amar, é preciso querer e provar. O amor envolve dedicação e renúncia, paciência e perseverança, trabalho e recompensa, alegria e tristeza, euforia e depressão. É algo envolvente e inebriante. O amor não é possessivo, ciumento, exclusivista, castrador, sufocante. Pelo contrário, o amor é libertário e altruísta. Não me refiro apenas ao amor erótico mas a todas as modalidades, inclusive as formas idealistas de amor à verdade, à justiça, à sabedoria, à humanidade, à natureza. O amor não pode ser cerceado em sua intensidade e abrangência. Ele não possui limites. Quanto mais se ama a mais coisas e pessoas, mais capacidade se tem de amar. E quanto mais se ama, mais se realiza e maior é a felicidade, mesmo que nem sempre seja correspondido. E certamente, é algo de que se possa contemplar em sua estética e fruir o máximo prazer que é amar o amor.
Veja isto:
http://www.ruckert.pro.br/blog/?p=4465
http://www.ruckert.pro.br/blog/?p=4440
http://www.ruckert.pro.br/blog/?p=4064
http://www.ruckert.pro.br/blog/?p=3851
http://www.ruckert.pro.br/blog/?page_id=1933
http://www.ruckert.pro.br/blog/?p=248
http://www.ruckert.pro.br/blog/index.php?s=amor+sexo

Se o clone e as pessoas próximas não tiverem memória sobre ele(clone), pois não terá havido uma vida(em memórias), ainda é antiético? Se sim, é por apenas ter um corpo Biótico?‎ Nagem Tannus

Acontece que um clone é feito a partir de um óvulo no qual se substitui o núcleo gamético por um somático e aquele óvulo terá que passar por toda uma gestação, dando origem a um ser com o contéudo genético daquele que doou o núcleo. Mas é outro indivíduo, que adquirirá outras memórias, em função da vida que experimentará. Não há como fazer o clone crescer sem memória. Assim, quando ele estiver adulto e se for transferir a memóprias dele. Como a personalidade está nos registros de memória, apagá-los e o mesmo que matar e pessoa. Se se fizesse um transplante de cérebro, em verdade o que se faria é um transplante de corpo, pois a pessoa é a do cérebro e não a do corpo.ória daquele que doou o núcleo do zigoto de que ele foi formado, será preciso apagar as memórias pr

Acha certo os psicopatas irem para a prisao? Ja que sao doentes deviam ser tratados, e nao punidos, certo?‎ Skyler

Sim, mas tratados com privação da liberdade, para proteger a sociedade. Ou seja, seriam presos, porque cometeram crimes, mas em uma prisão clínica. O sistema prisional tem três finalidades: proteger a sociedade, tirando o criminoso do convívio social, castigar o criminoso, para inibir os outros a cometerem crimes e recuperar o criminoso para a volta ao convívio social. No caso de psicopatas não haveria motivo para castigo. No entanto é preciso proteger a sociedade e recuperá-lo, por meio do tratamento.

Professor... Na sua opinião, qual foi o físico mais influente da história?‎ Marlon Di Gregori

Para mim foi Isaac Newton, pois foi quem derrubou, definitivamente, a concepção de que o mundo celestial fosse distinto do mundo terrestre, ou seja, que considerou que o Universo todo fosse uma entidade puramente natural. Essa mudança de concepção foi fundamental para o desenvolvimento de todas as ciências e da filosofia. Ouso dizer que foi o que possibilitou a Charles Darwin e seus colegas a formular a Teoria da Evolução e a Freud a considerar a mente como uma ocorrência orgânica, sem nenhuma menção à alma (no que foi vergonhosamente traído por Jung). Mesmo tendo se equivocado em algumas concepções, como a do espaço e tempo absolutos, foi quem elevou a Física ao estatuto de ciência fundamental, à qual se reduzem a Química, a Biologia, a Psicologia, a Sociologia e todas as demais.

Você se considera uma pessoa fria?‎ Jéssica Kirsch ✡

De modo nenhum. Nem sequer racional. Sou muito emotivo e intuitivo, além de dotado de grande sensibilidade. O fato de ser capaz de encaminhar um pensamento de modo racional e lógico não significa que pauto minhas decisões pela racionalidade, apesar de levá-la em consideração. Penso que os aspectos emocionais e intuitivos são tão importantes quanto dos lógicos e racionais nas decisões. E no tratamento com as pessoas, absolutamente não sou frio e impassível. Sou uma pessoa afável e carinhosa com todos. Mas não sou exuberante nem melodramático ou efusivo. Sou contido, gentil, cortês, bem educado, mas comedido. Em suma, temperado.

DESAFIO | O barbeiro é um homem da cidade que faz a barba de todos aqueles, e somente dos homens da cidade que não barbeiam a si mesmos. Tudo isso parece perfeitamente lógico, até que se coloca a questão paradoxal: Quem barbeia o barbeiro?‎ Numinoso

Esse paradoxo tem que ser colocado de outra forma: O Sultão obrigou o barbeiro, sob pena de morte, a fazer a barba de todo mundo que não pode fazer a barba a si mesmo e o proibiu, também sob pena de morte, a fazer a barba de quem possa fazê-la a si mesmo. A questão é: deve o barbeiro ser ou não condenado à morte, já que podendo fazer a barba a si mesmo, não poderia fazê-la e não podendo fazê-la, teria que fazê-la. Isso é um paradoxo insolúvel.

O que é o arco-íris? Como ele se forma?‎ aninha

Trata-se de um meteoro (ocorrência atmosférica) produzido pela refração e reflexão interna total da luz do Sol nas gotas de uma chuva ou de uma nuvem. Devido à diferença do índice de refração da água da gota para cada cor constituinte da luz branca, ao entrar na gota, se refletir por dentro e sair de volta, as diferentes cores se propagam com ângulos diferentes em relação à direção dos raios incidentes. Então, para um dado observador, o conjunto de raios de uma mesma cor fazem um arco em torno da luz que ilumina as gotas, como cores diferentes saem em ângulos diferentes, esses arcos ficam deslocados, um ao lado do outro, apresentando a configuração conhecida como "arco-íris".

Porque a dialética não foi a principal corrente filosófica, ao longo da história?‎ yasmin bernardes

Dialética não é uma corrente filosófica. E, apenas, uma forma de se desenvolver argumentações explicativas para uma série de fatos naturais e sociais. De fato, muitos podem ser explicados dialéticamente, mas nem todos. Não é possível estabelecer a dialética como uma forma universal de evolução natural e social. Numa outra concepção, dialética é, meramente, a arte de se argumentar em uma discussão.

O que diferencia a depressão de questões como tristeza e angústia?‎ Kirk Lazarus

Depressão é uma doença. Tristeza e angústia, como alegria, euforia, preocupação, ansiedade, tensão, são estados emocionais. A diferença é que esses estados são provocados por circunstâncias externas ou internas (lembranças) e desaparecem quando as razões se desfazem. A depressão, ao contrário, se manifesta como esses estados, especialmente angústia e tristeza que permanecem sem que as razões para eles se apresentem. Ela pode ser desencadeada por tais estados, mas evolui para uma situação patológica irreversível pela cessação dos agentes causadores. Requer uma terapia, até medicamentosa, para ser curada.

Não acha que é totalmente fora dos padrões de ética e moral, criar um clone só para dar continuidade a suas memórias? Ele tem a mesma constituição genética sua, mas é outra alma que o habita. Para mim seria como matar um ser humano da forma mais egoísta que possa existir.

Concordo plenamente. Não acho nada ético. Mas acho que será tecnicamente possível no futuro.

Já sofreu de depressão alguma vez na vida?‎ Kirk Lazarus

Não, mas de ansiedade, tensão, tristeza e angústia sim, muitas vezes.

Você não acredita que a tecnologia está tendo um uso negativo diante os jovens? Por exemplo, os adolescentes viciados em jogos?‎ Douglas Berna

Sim. A tecnologia, qualquer que seja, é boa em si, mesmo a bomba atômica. O que pode não ser bom é o uso que se dá a ela. Os jogos de computadores são bons instrumentos para o aprimoramento da inteligência, mas quando se vicia neles, torna-se algo ruim. O erro não está nos jogos e nem na tecnologia, mas no uso inadequado que está se fazendo dela. A questão é que o processo educativo formal não cuida de promover a educação do caráter e da personalidade, como seria obrigação de o fazer, desde a tenra infância e ao longo de toda a vida acadêmica. Isso teria que ser obrigatório e objeto de avaliação consequente, isto é, com possibilidade de reprovação do aluno por falta de ética ou de bons modos.

O seu humano pode criar uma "Vida após a morte?"‎ Douglas Berna

Atualmente, ainda não, mas poderá, no futuro, desenvolver algum modo de transferir o conteúdo todo da memória de uma pessoa para um clone dela ou para algum dispositivo biônico que passará a viver com o "eu" da pessoa que lhe transferiu a memória.

Professor, Qual a diferença entre nacionalismo e patriotismo?‎ Mad King

O nacionalismo tem uma característica econômica, isto é, valoriza os produtos nacionais em relação aos estrangeiros, protegendo a economia nacional da concorrência estrangeira mas, ao evitar a competitividade, acaba deixando os produtos nacionais com qualidade inferior. Já o patriotismo tem um caráter mais cívico e cultural, isto é, valoriza a cultura nacional e a afeição pelo próprio país, com certa rejeição aos valores estrangeiros e, até, certa xenofobia

Por que o céu é azul?‎ Bianca Belchior

Porque as moléculas do ar atmosférico espalham a luz e isso se dá, em razão do tamanho das moléculas, com as ondas de luz de tamanho correspondente à cor azul, enquanto o resto passa direto. Assim o azul é difundido para os lados. No por do Sol, ele é visto mais avermelhado porque sua luz tem que atravessar, rasante, uma espessura maior de atmosfera, causando mais retirada de azul, o que faz a sobra ficar mais avermelhada. As nuvens são brancas porque o tamanho, muito maior de suas gotículas, espalha as ondas de todas as cores e a mistura delas todas é o branco. As nuvens de tempestade são cinzas porque cada gota contém uma partícula de pó em seu interior.

Presenteá-lo com um livro deve ser muito difícil, devido a isso e a outros adventos, o senhor possui muitas cópias de algum livro ou de vários?‎ Numinoso

Não. Uma só de cada um. Mas, às vezes ganho, ou mesmo compro, sem saber, um livro que já tinha. Então doo o outro para uma biblioteca ou para uma pessoa amiga. Ou troco, na livraria.

Gosto tanto do senhor ! Adoro ler suas respostas. Responde ai pra miim : Oque o senhor tem a falar sobre a tecnologia ?‎ Sra. Katy ♪

Adoro tecnologia e acho um grande benefício para a humanidade. Só acho que é preciso um grande esforço social e econômico para que possa ser disponibilizada equitativamente pela população, bem como um esforço educativo para dela se fazer uso proveitoso e não prejudicial.

domingo, 29 de dezembro de 2013

http://bit.ly/15nKL7v Poderia me explicar o por quê da luz do Sol nesse eclipse se transformou nessas " ondas " ?‎ Dennis Siqueira

Isso é a "Coroa Solar", a região mais externa da atmosfera solar, normalmente não visível por causa do brilho muito maior do próprio Sol. Estende-se por alguns milhões de quilômetros acima da fotosfera, que é a mais externa camada opaca do Sol. A coroa é extremamente quente (cerca de dois milhões de kelvins) e ionizada, sendo a origem do "vento solar" que atinge os planetas.

Já quis montar uma biblioteca? Quais livros não poderiam faltar nela? Caso já tenha uma, diga os livros que quer comprar.‎ Dieniffer.

Não só quis como já montei ao longo de mais de meio século, sempre adquirindo, além de ter herdado. Hoje conto com mais de seis mil livros, dez mil revistas, quatro mil discos, mil vídeos, mais de 500 partituras, além de gravuras, mapas, e outros documentos, como apostilas, folhetos, teses etc. Isso me ocupa dois grandes cômodos de minha casa em que as estantes, do chão ao teto, com uns 18 metros de largura, são preenchidas com duas filas de livros, uma por trás da outra. A lista de compras já chega a uns 300, que vou comprando com um orçamento (incluindo livros, revistas e discos) de mais de mil reais por mês. Meu intento é montar uma ONG e disponibilizá-la ao povo, de graça, mas sem empréstimo, só com leitura local. Acredito que já gastei uns 3/5 de milhão de reais com ela.

O que me faz melhor em eu estudar filosofia?‎ GERALDO CÉLIO DA COSTA

O estudo de Filosofia tal qual é feito atualmente no Ensino Médio não contribui muito para isso, pois só se estuda a História da Filosofia e não se ensina a filosofar. Mas se você se dedicar à prática filosófica você verá que isso lhe tornará uma pessoa muito melhor. Porque você aprenderá a analisar a realidade, a examiná-la em profundidade e abrangência, a buscar as razões, os modos, as consequências, as propriedades, tudo a respeito de tudo. A questionar o que se está estabelecido, a contestar, se for o caso. A argumentar, a descobrir as falácias, e desnudar o cerne de tudo. Saberá comparar cada tema com outros correlatos mas de diferentes concepções. Tornará uma pessoa de mente aberta e livre de todo dogmatismo e preconceito. Saberá aceitar as diferenças, identificar o que seja válido ou inválido, inclusive nos raciocínios. E será treinado em descobrir a verdade e o que há de enganoso em toda proposição. Aprenderá a identificar a beleza e a pautar a vida pela prática do bem. Filosofar é uma atividade extremamente gratificante e prazerosa. A Filosofia é a mestra da vida e, muito mais do que qualquer religião, o conduzirá pela via da virtude e da sabedoria a encarar a própria vida e sua relação com o mundo e as outras pessoas de uma forma madura, consistente, significativa, proveitosa. Assim você será levado a conquistar a paz de espírito e a felicidade que é a culminância de uma vida plena e grandiosa. Você se tornará uma pessoa cuja existência será valiosa para o mundo. Você fará diferença e sua luz iluminará o caminho dos outros. Sem apelar para nenhuma entidade superior e sem o uso de nenhuma prática esotérica e nenhuma encenação litúrgica. Simplesmente por suas palavras verdadeiras e serenas, por sua atitude, por seu exemplo, por sua vida.

O senhor já chegou a ler algum livro do escritor Érico Veríssimo? Se a resposta for afirmativa, poderia dar a sua opinião sobre ele? Obrigada. (Sou grande fã desse escritor)‎ Olivier

Sim, já li "Olhai os Lírios do Campo" e "Incidente em Antares". Excelente escritor. Gostei muito.

Certamente se a evolução não foi guiada por um Deus, nós teriamos evoluído com a mente focada na sobrevivência, e não na razão. Como vc explica isso?‎ Tokoo -♔

A evolução não é focada na razão. Ela acontece em duas etapas. A primeira é totalmente aleatória, que são as mutações. A segunda, a seleção natural, faz com que, ao longo do tempo, e um tempo bem longo, as variações que melhor contribuem para o sucesso adaptativo, que consiste na sobrevivência até a geração de prole, sejam favorecidas e passem a se tornar majoritárias na coletividade, até que as menos bem adaptadas desapareçam. Se a modificação for de grande monta, após muitas gerações, vai se caracterizar uma nova espécie. No caso da evolução humana, foi, justamente, o aprimoramento da racionalidade, que favoreceu o sucesso reprodutivo, pois os humanos, com o seu pensamento, urdiram defesas contra os predadores e meios de obter mais alimentos e transar mais. Há outros fatores, que não têm nada a ver com racionalidade, que também foram importantes no processo evolutivo, como um sistema imunológico mais eficaz contra os parasitas patogênicos internos. Nossa mente, mesmo com a racionalidade, é focada na sobrevivência. Em verdade só se filosofa depois que se enche a barriga. E toda a nossa racionalidade tem por meta principal encher a barriga e fazer sexo. O resto existe em função disso, mesmo que possa não parecer e que, excepcionalmente, possa haver quem se compraza em passar fome ou se abster de sexo. Deus não tem nada a ver com isso. A evolução não é guiada por nada. Não tem o menor objetivo.

Se a física e o acaso guiam a natureza, o natural seria que vc não tivesse controle dos próprios pensamentos. Então por que você continua se questionando sobre o Universo?‎ Tokoo -♔

De modo nenhum. Para começar o acaso não guia nada. Se algo acontece por acaso, não é guiado, é fortuito. E o fato das ocorrências mentais serem naturais não exclui a possibilidade do libre arbítrio nem das escolhas, das contestações, exatamente porque a natureza não é determinística. Essa característica intrínseca dos eventos naturais, o não determinismo e a não necessidade de causalidade é que permitem toda a margem de manobra mental, a existência do desejo e da vontade. Se o mundo fosse como o concebia Laplace, ninguém teria nenhuma liberdade. Ainda bem que não é assim.

Professor, eu to em um caso sério de ciúmes do meu namorado (a gente namora a distância) eu chego a chorar quando ele vê vídeo de meninas americanas cantando, por exemplo, não suporto nenhuma menina perto dele e por isso proibi ele de falar com qualquer menina.. oq eu faço? to ficando louca :(‎ .

Você precisa se convencer que, por mais que você ame seu namorado, você não é dona dele. Ele é um ser humano inteiramente livre. Amor não pode ser uma obrigação. Ele deve te amar porque é o que ele quer. Isso não o impede de apreciar outras meninas e, até, gostar de alguma também. Amor não é exclusivista. Isso é o que você tem que entender e assimilar. Seu ciúme não é prova de amor por ele e sim prova de seu egoísmo em querer que ele ame só a você. Pense muito nisso e ame de forma plena e liberta de todo sentimento de posse e controle sobre a pessoa amada. Esse é que é o verdadeiro amor. Do mesmo modo que você também pode apreciar outros rapazes e até gostar de mais de um.

Ernesto eu já te sigo há um bom tempo e adoro tuas respostas. Será que podes me recomendar algumas obras de Fiódor Dostoiévski? Vou frequentemente à livraria e fico em duvida do que comprar... Se puder me ajudar, obrigada.‎ Bela

De Dostoiévski eu recomendo, nesta ordem: Crime e Castigo, Os Irmãos Karamázov e O Idiota. Também gosto muito do conto "Noites Brancas".

Ernesto eu já te sigo há um bom tempo e adoro tuas respostas. Será que podes me recomendar algumas obras de Fiódor Dostoiévski? Vou frequentemente à livraria e fico em duvida do que comprar... Se puder me ajudar, obrigada.‎ Bela

De Dostoiévski eu recomendo, nesta ordem: Crime e Castigo, Os Irmãos Karamázov e O Idiota. Também gosto muito do conto "Noites Brancas".

Será que ainda irá demorar muito para a profissão de Cientista seja regulamentada no Brasil? Eu não entendo muito bem como isso funciona, só é permitido fazer pesquisas se estiver vinculado à uma instituição/Universidade?‎ Mayara Góes

Claro que não. Qualquer um pode fazer pesquisa, mesmo particularmente. E não precisa ter diploma de nada. E eu acho que é bom que continue assim. Sou contra a regulamentação de profissão de cientista. Acho um absurdo se exigir formação específica para investigar, descobrir ou inventar qualquer coisa. É claro que quem tem a formação e está ligado a uma instituição tem muito mais condições de levar adiante uma pesquisa. Mas não pode ser uma exigência. Aliás sou contra reserva de mercado para quase tudo, exceto para profissões que envolvam responsabilidade civil, como médico. Até advogar eu acho que se poderia sem diploma nem exame de ordem. Lecionar então, nem se fala. Sei de muita gente que não é licenciado em magistério e que é professor muito melhor do que os licenciados. O que importa é o conhecimento e a competência e não o diploma. Acho um absurdo se ter que dar um lugar de professor a um licenciado medíocre e negá-lo a um engenheiro, médico, advogado ou outra pessoa brilhante, inclusive didaticamente, só porque não tem o diploma. Isso é um desserviço à educação. Quem não tem competência não pode se estabelecer. O mérito tem que prevalecer sobre a legalidade.

Ernesto, a lei Maria da Penha não ajudou. http://g1.globo.com/brasil/noticia/2013/09/lei-maria-da-penha-nao-reduziu-morte-de-mulheres-por-violencia-diz-ipea.html‎ Q.

Pois é, tá vendo? Não precisa de lei específica para isso. É só aplicar a lei geral. Mas nem a geral nem a específica adiantam. Elas são frouxas e a polícia também, nesse caso. A cultura machista ainda impera. Só com muita educação isso vai mudar e demora. Demora muito.

O mundo, o Sistema Solar e nós insetos espaciais haha! todos nós vamos virar poeiras cósmicas hahaha Lixos espaciais sabe porque? Um corpo celeste e uma estrela 10x Maior que Júpiter está em nosso "Sistema Solar" e o nosso querido (Governo) esconde de "Nós" para não causar pânico na população haha.‎ felipinho

Você acha que uma estrela 10 vezes maior do que Júpiter que estivesse dentro do Sistema Solar não seria vista como o objeto mais brilhante do céu, depois do Sol? Cadê ela? Isso seria tão notável que não se poderia esconder. E depois, algo tão grande seria detectado muito antes de chegar ao sistema solar em todo o mundo, por todos os astrônomos, inclusive amadores. Isso não conseguiria ficar abafado. Esse negócio de Nibiru, Planeta X ou Hercobulos é puro embuste.

Olá Ernesto, o que você acha da trilogia 50 tons de cinza? Já leu? gostou?‎ Eu Conto Depois

Não li. Pelos comentários a respeito, não sei se é o tipo de literatura que apreciarei. Não porque tenha prevenção contra enredos eróticos, mas porque tenho prevenção contra romances de baixa qualidade literária. Ainda prefiro clássicos, como Dostoievsky, Victor Hugo, Charles Dickens, Tolstoi, Eça de Queirós, Machado de Assis. Mas gostei de Tolkien, como gosto de Asimov e Clarke. Gosto de LeGuin, Lessing, Buck. Mas não apreciei muito Dan Brown, Jô Soares. Quero ler o George Martin.

Sobre a violência contra as mulheres, qual a sua opinião? A lei Maria da Penha ajudou a diminuir a violência ou não?‎ Biblioteca

Violência não pode ser perpetrada contra ninguém. Para mim não precisava haver essa lei. Bastava a lei normal de que violência é crime contra quem quer que seja. Mas... infelizmente no Brasil, as leis comuns não valem. Só se for uma especial. Quanto a ela ter ajudado a combater a violência contra mulheres, não conheço os levantamentos estatísticos a respeito, mas penso que deva ter ajudado, já que antes não se aplicava a lei ordinária sobre o assunto, pois se achava que maridos tinham direito de bater em suas mulheres.

O que o senhor acha de pessoas tatuadas ? ( tenho tatuagens )

Depende da tatuagem e da localização. Não gosto de tatuagens muito extensas. Acho que ficam bonitas, em mulheres, no ombro, entre os rins ou abaixo do umbigo, pequenas e singelas (flores, borboletas, alguns signos) e de cores claras. Em homens, apenas no braço, se for o caso. Uma águia ou um signo. Heraldico, por exemplo. Mas não dragões. Não faria nenhuma em mim mesmo.

Ernesto, qual a origem e a forma como a luz chega aos olhos?‎ Biblioteca

Luz é a modalidade de onda eletromagnética na faixa de frequência capaz de excitar as células sensíveis da retina. Esses artigos lhe darão uma boa ideia do que seja luz:
http://pt.wikipedia.org/wiki/Luz
http://pt.wikipedia.org/wiki/Cor
http://pt.wikipedia.org/wiki/Olho
http://pt.wikipedia.org/wiki/Olho_humano
http://pt.wikipedia.org/wiki/Vis%C3%A3o

O que você acha do método de ensino das escolas do ensino médio (tanto particulares quanto públicas)?‎ Gi

Completamente ultrapassado. Atrasado trezentos anos. A moderna pedagogia não contempla a existência de turmas, de salas, de aulas, de séries, de provas, nada disso. A escola é de horário integral, mas os alunos podem chegar e sair a hora que quiserem. Os professores são todos em dedicação exclusiva e ficam por conta da escola. Toda ela são bibliotecas, laboratórios, oficinas, computadores, mesas de estudo. O aprendizado é feito por projetos de estudo e pesquisa multidisciplinares feitos em grupos de três e defendidos perante uma banca, que aprova ou desaprova, sem nota. Cumpridos os projetos exigidos, se tem o grau feito. A ordem dos projetos é da escolha do aluno, atendidos os pré-requisitos. Não há aprovação de série. Os grupos podem ser variáveis e terem alunos de diferentes adiantamentos. Aprende-se várias coisas úteis, como cozinhar, costurar, lavar, passar, dirigir, consertar instalações elétricas e hidráulicas, bem como aparelhos diversos, plantar e cultivar verduras, frutas, tratar de animais, controlar as finanças. Pintura, música, dança, teatro, poesia. Prática de vários esportes. Tudo de forma engajada em projetos reais e interagindo com a comunidade. O tempo para a conclusão do grau vai depender de cada um. O progresso é individualizado. Isso é que é uma forma prazerosa e eficaz de se aprender para a vida.

me tira uma dúvida por favor, se uma pessoa for mulher é ela for ateu o certo é falar "atéia " ou ateu pode fala para qualquer sexo ?‎ Raufa

O feminino de ateu é atéia mesmo.

Acompanha a linha de raciocínio que de, certa forma, grande parte das religiões foi criada com o intuito de ''escravizar'' e colocar um pouco de ordem na vida dos homens?‎ Arthur

Não é que elas foram estabelecidas para isso, mas que os detentores do poder se aproveitaram das crenças e se uniram aos sacerdotes para fazer uso das religiões para controlar o povo e induzi-lo a acatar seus desígnios. A dita "ordem" não é uma ordem harmônica, que brota do acatamento democrático de um padrão de comportamento que permita a tranquilidade social, mas uma ordem imposta, que não perturbe as benesses das classes favorecidas. Por isso é que o anarquismo não aceita as religiões, uma vez que elas são hierárquicas e completamente avessas ao libertarianismo. Em suas origens muitas, como o próprio cristianismo, surgiu como uma rebeldia em relação aos poderes estabelecidos. Mais tarde, contudo, o poder se uniu à religião cristã, com Constantino e ela, de oprimida passou a opressora.

Professor como posso ''entender'' melhor a física ? Por mais que eu tente não consigo resolver nenhum exercício sequer .‎ Julia Gimenez

Há dois aspectos importantes na Física. O conceitual e o operacional. Para se dar bem no segundo, isto é, resolver problemas, é preciso se ter um bom conhecimento do primeiro. Alguns professores e, mesmo, alguns livros, não dão muita atenção à parte conceitual, fixando-se mais na prática. Mas a prática só deslancha quando a teoria estiver bem assimilada. Não sei qual o livro que você estuda, ou se é apostila, mas há um muito bom, chamado "Física Conceitual", de Paul Hewitt (Editora Bookman) que te dá uma excelente base conceitual, necessária para identificar os fenômenos presentes em uma questão e as fórmulas para resolvê-la, inclusive os casos em que elas não se aplicam. De qualquer modo, para entender bem de Física (ou de outro assunto) é importante gostar dela. É preciso querer saber e não só apenas o que é ensinado. Indo além, sabe-se também o que se exige. Quem sabe demais, sabe o menos. Esse é um bom livro para aprender a gostar de Física. Ele pode ser baixado de graça pela internet: http://www.degracaemaisgostoso.org/download-fisica-conceitual-paul-g-hewitt.html

essa garota ai esta certo o ask e so para pessoas de 19 anos para baixo não chamando o senhor de velho mais isso aki não e para você não agora errando ela de chama de velho errando você faz ask , não quero sabe mais de nada a hora e assim todo mundo pode toda af's :/‎ Lolo _|_

Discordo totalmente de você, não só do conteúdo de sua postagem como também do português que você usa. Esta é uma comunidade livre, da qual todos que quiserem podem participar, independentemente da idade e de qualquer outra característica, como escolaridade, inteligência, religião, ideologia, sexo, etnia ou o que for. Se estou sendo requisitado por perguntas e minhas respostas têm sido apreciadas, acho que meu lugar aqui é proveitoso, mesmo para a juventude, por quem nutro muito carinho, respeito e admiração por suas atitudes corajosas de enfrentamento de preconceitos, de rejeição de estereótipos, de desejo de mudança do mundo e de luta por grandes melhorias. Busco encorajá-la e dissipar suas indecisões e angústias no que posso ser capaz. E também, com os jovens, aprendo muita coisa, pois nunca se é velho demais para aprender e nem se pode dizer que já se tenha sabedoria suficiente. Há que se reconhecer tanto o valor quanto as limitações e estar sempre aberto a novas ideias. Não vejo porque limitar a participação a 19 anos.

riticulo isso não e para velho nãoo‎ #yolo. ♥ *

Estou respondendo esta questão só para mostrar como existe preconceito até na juventude. Porque este Ask não seria para adultos também? Pelo que posso depreender do número de perguntas que me dirigem (7801, das quais 3528 já respondidas em quatro meses) e do número de "likes" que já ganhei (33352) acho que boa parte da juventude está gostando de minha participação. Além do fato de que eu vim para cá a convide, justamente, de jovens. Não vejo, assim, que esteja eu fazendo nenhum papel ridículo aqui.

Poderia me dar alguma dica pra eu ficar mais à vontade quando tiver que falar em público?‎ Kirk Lazarus

A principal diga é: perca a vergonha. Isso mesmo. O grande problema é a pessoa ficar achando que vai ser ridícula e se travar para não dar vexame. É preciso fazer um treinamento perante uma platéia de amigos para perder o medo de pagar mico. Não ligue para dar vexame. Dê o vexame tranquilamente, passe por cima e continue. Para isso é importante esse treino. Aos poucos a pessoa vai se destravando e adquire traquejo. Vergonha é um grande entrave. É preciso desinibir-se. Mas nunca se vai conseguir isso sem ousar. É como andar de bicicleta. Só vai aprender depois de alguns tombos.

Vc e contra o catolicismo?‎ lukinha

Não especificamente contra o catolicismo, mas contra qualquer religião, seja cristã ortodoxa, protestante, islâmica, hinduísta, budista, judaica, xintoísta, pagã, zoroastrista, kemetista, siquista, jainista, espírita, umbandista, candomblesista ou qualquer que seja. Penso que as religiões promovem um engessamento da mente, uma atitude antolhista. Impedem o livre pensamento, a busca despreconcebida da verdade, a expressão da dúvida. Acabam incentivando uma animosidade entre elas e, com isso, atrapalham uma verdadeira solidariedade e um completo congraçamento da humanidade toda. Mesmo que uma pessoa acredite na existência de divindades, de espíritos e de uma alma, é melhor que ela não atrele sua crença a nenhuma religião. Não que elas não promovam algo de bom, pois o fazem. Mas tudo o que fazem de bom pode ser feito sem elas, como a caridade e o consolo. O significado que podem dar à vida também pode ser dado por uma atitude filosófica. Aliás, para mim, é a filosofia que tem que substituir a religiosidade na vida humana. Para tal precisa ser cultivada desde a infância, na escola, no lugar das aulas de catecismo ou das escolas dominicais protestantes, bem como dos estudos bíblicos, corâmicos, torálicos, talmúdicos, védicos, avestas, pitakas e outros. Pela Filosofia a pessoa consegue entender a vida e pautá-la eticamente, encontrando significado para ela sem a necessidade de se filiar a nenhuma religião. As religiões consomem muita energia, tempo, dinheiro e outros recursos, que poderiam ser muito melhor usados para coisas mais proveitosas. Templos poderiam se transformar em centros de estudos, bibliotecas, salas de teatro, museus e outras finalidades educativas e culturais. O catolicismo, especialmente, é uma mega-instituição super complexa, que consome, só na estrutura administrativa e burocrática, sem mencionar sua pompa supérflua, um volume de recursos colossal, que, se aplicado judiciosamente, poderia contribuir imensamente para a erradicação da fome, da ignorância, da doença e de outros males. Realmente, a extinção de todas elas seria um imenso benefício para a humanidade.

O cientista tem obrigação de divulgar suas descobertas ao domínio publico ou tem o direito de guardar o conhecimento para si?‎ Bruno Ribeiro

Se ele é financiado por alguma instituição pública, o resultado de suas pesquisas tem que ser público. Se ele é financiado por particulares, que pode ser ele mesmo, então o resultado é propriedade de quem financia, que pode divulgar ou não. No meu entendimento, se a pesquisa for básica e não aplicada, existe um compromisso ético de que seja divulgada, já que, diretamente, não produz retorno econômico. Isso acontece, por exemplo, com pesquisa em cosmologia e em física de partículas elementares. O conhecimento advindo é economicamente inútil.

Professor poderia dar uma explicação diferente sobre Efeito Doppler??‎ Nagem Tannus

Diferente como? Efeito Doppler é o fenômeno de uma onda apresentar uma frequência diferente na emissão e na recepção como resultado do movimento da fonte em relação ao receptor, bem como do meio de propagação, se houver. A análise é diferente para ondas mecânicas, como o som e eletromagnéticas, como a luz, já que a velocidade de propagação desta é a mesma em todos os referenciais. Isso não é um assunto simples e a análise requer esquemas e algumas páginas de explicação. Sugiro visitar estes sites:
http://pt.wikipedia.org/wiki/Efeito_Doppler
http://www.sofisica.com.br/conteudos/Ondulatoria/Acustica/doppler.php

Os proto-humanos e os humanos primordiais eram todos ateus – como os são os grandes primatas atuais –, certo? Características rústicas de alguma forma religiosa são encontradas apenas no Homem ou também nos seus antecessores ("Homo erectus", p. ex.)? E de quando a 1º manifestação religiosa é datada?‎ Daniel

Culturas pré-humanas enterravam seus mortos. Isso é considerado uma manifestação de crença em uma vida após a morte. Tal crença, normalmente, é associada à crença de divindades. Não sei precisar a datação desses achados.

Acho que vc não acredita em alienígenas rsrs. Mas vou perguntar mesmo assim. Acredita na teoria dos alienígenas do passado ? Se não , acredita que teria alguma forma de os antigos levantarem aquelas pedras de granito sem tecnologia ? Tipo , hoje com toda nossa tecnologia ainda temos dificuldades pra‎ Unbroken

Não acho que alienígena nenhuma tenha vindo à Terra e interferido na construção das pirâmides do Egito, por exemplo. Claro que eles poderiam fazê-las com métodos disponíveis em sua época. E hoje também. A questão é que elas levaram décadas para serem construídas e consumiram o trabalho de milhares de homens. Atualmente não se tem dinheiro e nem disposição para fazer algo assim. Mas se se quiser, claro que se consegue. Mesmo as catedrais góticas da Idade Média, hoje não são feitas, pois sua construção levava alguns séculos. Atualmente não se tem paciência para isso. No entanto há um caso assim: A Basílica da Sagrada Família, em Barcelona.

Professor gostaria muito da sua opinião, como o senhor sou ateu, porém desde que tinha 8 anos, me despeço da minha mãe dizendo "Tchau mãe, fica com deus", é um costume que não consigo largar, isso é um mau indício em relação ao meu instinto natural de ateu?‎ Gabriel Fernandes

Não tem problema nenhum. É apenas uma figura de linguagem, como "Nossa Senhora!", "Credo!", "Deus me livre!", "Se Deus quiser!". Todas elas já se incorporaram ao linguajar, independentemente de se considerar que Deus e Nossa Senhora, de fato, existam ou não. Mesmo quem creia em Deus, ao dizer essas expressões, não está pensando, especificamente, no sentido literal delas. Só uma observação. Quem foi criado em um ambiente familiar de crença em Deus e, posteriormente, se tornou ateu, seu ateísmo não é instintivo e sim racional. Uma consequência de suas reflexões, ponderações e conclusões. Só se pode dizer que uma pessoa seja instintivamente ateia, se, em sua criação, desde a infância, ela nunca foi apresentada ao conceito de Deus. Isso é quase impossível para uma pessoa civilizada, mesmo que sua família seja ateia, pois a ideia de Deus está difundida por toda a sociedade, qualquer que seja ela, não apenas a cristã.

POR QUE FISICA É TÃO COMPLICADO E TÃO IMPORTANTE EM NOSSO DIA DIA‎ Paulo Ricardo Fogaça

Nada é complicado se você souber. E você ganha conhecimento por etapas, galgando degrau por degrau. Claro que, se você, desde os primeiros anos do Ensino Fundamental, nunca se interessou por ciências, nunca teve aquela grande curiosidade para saber como o mundo funciona, nunca foi daqueles que não se contentava apenas com o que o professor ensinava, mas queria saber muito mais, então você não será fascinado pela Física, e, portanto, a achará difícil. Qualquer assunto pelo qual você tenha uma grande atração, se tornará fácil para você, porque você o estudará com muito gosto. A Física é importante porque é o fundamento de todas as aplicações tecnológicas. Engenharia é Física Aplicada. Daí a necessidade cada vez maior de físicos e engenheiros para sustentar a continuidade do progresso, que, cada vez mais, tem um componente muito grande no aspecto tecnológico.

O SR ACREDITA QUE PODEREMOS NUM FUTURO PRÓX FAZERMOS VIAGENS AO ESPAÇO TIPO ÔNIBUS ESPACIAIS PARA 30 PAGANTES‎ Paulo Ricardo Fogaça

Claro que sim. Novamente não é uma questão de acreditar, mas, simplesmente, de fazer projeções com base nas tendências do desenvolvimento tecnológico e das demandas comerciais (por favor, use caixa baixa (minúsculas)).

como cientista, de onde surgiu tudo, me responda exatamente, de onde surgiu o hidrogenio e os componentes atomicos do hidrogenio, e os componente dos componentes atomicos, e mais menos ainda mais divisoes, e mais divisoes atomicas possiveis, de onde surgiu o minimo do atomo e da luz‎ Waldo Damha Junior

O hidrogênio surgiu no próprio Big Bang, logo depois do surgimento dos quarks e dos léptons e suas antipartículas. Estes surgiram da quantização do campo primordial nos primeiros instantes do Universo. A luz, que é uma radiação eletromagnética, surgiu logo que as primeiras partículas e antipartículas se aniquilaram, transformando-se em dois fótons. Tudo isso é modelado por teorias bem fundamentadas. Só não se sabe como surgiu o campo primordial que começou a ter o seu espaço expandindo-se, fato que, exatamente, consiste no Big Bang. Ainda não se tem uma explicação para tal surgimento. Isso não significa que a explicação não será obtida e nem que a explicação seja uma interveniência sobrenatural. Existem propostas, como a teoria das branas, mas ela não foi confirmada e, particularmente, não penso que seja correta. Para um bom apanhado do início do Universo, sugiro a leitura do livro:
http://www.amazon.com/The-First-Three-Minutes-Universe/dp/0465024378

Ola Ernesto,vi algumas respostas suas no ask da minha irma caçula,e admiro a sua capacidade de explicação. Pelo fato de acha lo uma pessoa com o intelecto capaz de saciar a minha curiosidade,pergunto uma coisa que sempre busquei saber,quem foi Lilith?Ela era de fato a primeira esposa de Adão? obriga‎ Emy Criss

Adão, Eva e Lilith são figuras lendárias. Lilith aparece na mitologia babilônica e judaica. Esse artigo esclarece a questão: http://pt.wikipedia.org/wiki/Lilith.

Boa tarde Ernesto! Gostaria de saber qual a importância, na sua opinião, do estudo da História?‎ Renato Mansotti

O primeiro e maior de todos é, simplesmente, satisfazer a curiosidade sobre o que aconteceu no mundo no passado. Mesmo que não sirva para mais nada, isso já justifica o investimento na investigação histórica. Além disso, o conhecimento de história ajuda na tomada de decisões políticas, econômicas e sociais, por se ter conhecimento das consequências dos atos de diversos tipos que se deram. Claro que isso não é determinante e nem garantido de que se dará, de novo, do mesmo modo. Mas é uma indicativa bem forte.

O'que acha do casamento homossexual?‎ Lαurα Liмα Susviєłα

Algo perfeitamente normal e legítimo, como o casamento heterossexual. Só que, para mim, nem um nem outro são necessários para nada.

tenho mts amigos mas ando apenas com 1, ele tem outro amigo q ele anda junto, qdo esse amigo n tá no colegio ele so anda comigo, qdo esse amigo ta ele me deixa um pouco de lado, sozinho, falei com ele e parei d falar com ele, agora ele puxa conversa pra eu voltar a amizade como era antes, oq fazer ?‎ claudineia

O que eu acho é que não se pode exigir nada de ninguém. Se você gosta de seu amigo e da companhia dele, frua isso com o máximo prazer e não faça questão de que ele se dedique exclusivamente a você. Se ele gosta também de outra pessoa, deixe ele curtir essa amizade também, do modo como ele prefere. Se ele retorna para você é porque também gosta de você. O fato de ele gostar de outro não diminui a afeição que ele lhe dedica. Mas você poderia conversar com ele e dizer que gostaria de que vocês três pudessem compartilhar a companhia de todos. Mas não diga isso em tom de reclamação e nem de forma choramingosa. É importante, em todo relacionamento, seja de amizade, seja amoroso, que ninguém se comporte de modo servil. É preciso se impor com altanaria mas sem soberba, isto é, de forma assertiva. Para tal há que se imbuir da convicção de que, de fato, por mais que se deseje ter a amizade e o amor do outro, não se é escravo disso, ou seja, tem-se personalidade suficiente para bastar-se a si mesmo, por mais que se compraza com a amizade e o amor de alguém.

Muitas pessoas acham cruel duvidar do amor de outra pessoa, independente se ela não agiu conforme as suas expectativas. Você também pensa assim? Por quê?

Duvidar é um direito de todos, a respeito de qualquer assunto. Todavia não acho que ninguém deva ter nenhuma expectativa sobre o comportamento dos outros, uma vez que todos são livres para pensar, sentir e agir como queiram, desde que as ações não sejam prejudiciais. Mas não amar não é uma ação propriamente prejudicial. Não se pode exigir amor de ninguém. Apenas desejar e esperar. Outro aspecto que também tem que ser considerado é que, muitas vezes, pensa-se que algum sentimento ou ação signifique falta de amor quando não se trata disso. Refiro-me, especialmente, ao caso de uma pessoa que nos ame também amar a outra pessoa. Isso não significa, em absoluto, que não nos ame. É perfeitamente normal ao ser humano amar a mais de uma outra pessoa. As pessoas seriam muito mais felizes se esse fato fosse aceito como normal. Assim não se precisaria renunciar a nenhum amor em favor de outro, mas se poderia manter ambos (ou até mais de dois) com pleno conhecimento e consentimento dos envolvidos. No caso em tela, o que quero dizer é que amar a outro não significa deixar de amar ao primeiro. Daí, realmente, ser uma temeridade achar que não se é amado por não se ser exclusivamente amado. Nesse caso, duvidar do amor é uma crueldade sim. Mas há casos em que a dúvida pode ser legítima, se o comportamento for, por exemplo, de indiferença, implicância, rejeição, irritação, contestação ou algo do tipo.

Que presidente brasileiro você ressuscitaria?‎ Helysa.

Juscelino Kubitschek de Oliveira.

Muitos dizem que o ramo das ciências biológicas não da futuro, qual sua opinião sobre isso?‎ Elaine

Claro que dá futuro. Existem grandes projetos de desenvolvimento de interfaces biônicas, por exemplo. Além do mais, a todo momento estão sendo descobertas novas espécies e os trabalhos de mapeamento biológico de biotas diversas está em pleno desenvolvimento. Sem falar da área ecológica, em que o trabalho do biólogo é essencial para a definição de medidas a serem tomadas pela Engenharia Ambiental tanto para a preservação ambiental quanto para o encaminhamento de projetos de desenvolvimento sustentável. E, além de tudo, há o campo do magistério. Tenho uma sobrinha que é doutora em ecologia pela UFMG e trabalhou na Central de Furnas, fazendo o controle do ecossistema da represa, e agora é professora da Universidade Católica de Itajubá.

O Senhor acredita em sorte?‎ Rhas Vinícius #CrazyMotherFuck

Claro que não. Não existe tal tipo de coisa. O desenrolar da vida de uma pessoa se dá em função de suas ações, suas decisões e, principlamente, das coincidências e acasos que se apresentam. Mas isto não é sorte, no sentido de ser alguma predisposição para que os eventos ocorram favoravelmente à pessoa. O que se considera como sendo sorte é, meramente, um acaso ou uma coincidência.

Você defende o que acredita? De que maneira?‎ Jéssica Kirsch ✡

Claro. Por palavras e por ações. Pela condução da minha vida. Pela divulgação de minhas ideias em minhas aulas, no que escrevo e pela internet. O que acredito está sumulado aqui:
http://www.ruckert.pro.br/blog/?page_id=2454

O que acha do preconceito existente hoje em dia com pessoas ateus ? Sou ateu e as vezes acabo sofrendo com isso !‎ Victória Miotto

De fato, há muita gente que considera que pessoas ateias sejam "do mal", isto é, imorais, cruéis, desonestas, niilistas ou tudo de ruim. Não é nada disso. Exceto por não crerem na existência de deuses, no mais, são como todas as outras pessoas, não havendo preferência por nenhum tipo de comportamento ético por ser ateu. Assim tanto podem haver pessoas ateias que correspondam a esse estereótipo quanto que sejam pessoas justas, bondosas, éticas de do bem. Se este é o seu caso, você tem que deixar isso patente, para que todos vejam como se equivocam. Mas, como acontece com todo mundo que tenha uma postura incomum, tem que se possuir uma personalidade muito marcante para se ter a sobranceria capaz de não se amofinar com o conceito dos outros, por, justamente, se estar bem convicto de sua concepção. Tal preconceito demonstra que se trata de pessoa mesquinha e sem abertura de mente para conceber que outros podem ter valor sem serem filiados a suas próprias concepções religiosas ou de outra ordem.

O que você acha das pessoas que, só por serem ateus se acham superiores, e, donos da verdade?‎ Joel Moreira.

Esta é uma postura ridícula e prepotente, não condizente com a sabedoria que toda pessoa deve possuir. E ela mostra que ninguém pode ser considerado dono da verdade, seja qual for a sua concepção. O que cada um pode ter é a convicção de ser possuidor da verdade, mas nunca a certeza. E a sabedoria consiste em sempre se duvidar de que se esteja certo, seja-se ateu ou crente de qualquer crença. O dogmatismo, entre crentes, até que pode ser desculpado, por serem as crenças dogmáticas em sua essência. Mas o ateísmo possui o ceticismo como um de seus pilares. Logo o ateu é, justamente, quem não pode ter certeza de que Deus não exista. Então ele assume essa inexistência por falta de evidências, comprovações e, até, indícios da existência. Sempre aberto, contudo, a rever sua postura, uma vez demonstrado que possa estar errado. É fato que há uma correlação entre ateísmo e cultura, especialmente científica. Isso, contudo, não garante que, por se ser ateu, se seja culto ou inteligente, nem que todo que seja culto e inteligente seja ateu. Quem se jacta de sua cultura e inteligência, fundamentando-se em seu ateísmo, comete grande equívoco, além de demonstrar um esnobismo completamente injustificado e de suprema deselegância.

Acreditas que existe o amor verdadeiro? E que o ser humano é capaz de possuir esse sentimento?‎ Çabryna

Claro que sim. Existe e as pessoas o vivenciam. Muitas pessoas. Há quem não ame em absoluto, ou quem finja que ama, mas a maior parte ama de modo sincero. Ou seja vive verdadeiramente um sentimento e tudo o que o amor significa. Qualquer um que reflita suficientemente sobre o significado da vida pode concluir que o amor é algo essencial à vida. Amor é um sentir, um pensar, um desejar, um querer e um agir, isto é, uma emoção, um sentimento, um apetite, uma volição e uma ação. Mas também é uma intelecção, refletida, consentida, explicitada e assumida. Esse complexo de fatos psíquicos caracteriza o amor em todos os planos, piedade, compaixão, solidariedade, afeição, amizade, amor platônico, erotismo. Amor filial, maternal, paternal, fraternal, conjugal, idealista. A intensidade e a sequência em que eles aparecem pode variar. O amor sempre emociona, enternece, e envolve um desejo de zelo, cuidado e proteção da coisa amada, bem como um desejo de reciprocidade. Mas o amor só se realiza quando é expresso em vontade e essa vontade em ação. Não basta sentir e desejar para amar, é preciso querer e provar. O amor envolve dedicação e renúncia, paciência e perseverança, trabalho e recompensa, alegria e tristeza, euforia e depressão. É algo envolvente e inebriante. O amor não é possessivo, ciumento, exclusivista, castrador, sufocante. Pelo contrário, o amor é libertário e altruísta. Não me refiro apenas ao amor erótico mas a todas as modalidades, inclusive as formas idealistas de amor à verdade, à justiça, à sabedoria, à humanidade, à natureza. O amor não pode ser cerceado em sua intensidade e abrangência. Ele não possui limites. Quanto mais se ama a mais coisas e pessoas, mais capacidade se tem de amar. E quanto mais se ama, mais se realiza e maior é a felicidade, mesmo que nem sempre seja correspondido. E certamente, é algo de que se possa contemplar em sua estética e fruir o máximo prazer que é amar o amor.

E A VIDA EM MARTE É PROVÁVEL OU MAIS UM LANCE QUE OS POTENTES NOS LANÇAM PARA OBSCURECER CERTOS MANEJOS QUE NÃO SERIAM DE INTERESSE DELES QUE A MAIORIA SOUBESSEM‎ Paulo Ricardo Fogaça

Penso que não. Imagino que já possa ter havido, mas não mais hoje. Suas condições são bem inóspitas. Todavia não vejo nada de lance para ocultar fato nenhum.

O HOMEM CHEGARÁ A DESCOBERTA DE TODOS OS MALES E CONSEGUIRÁ EXTINGUIR A FOME‎ Paulo Ricardo Fogaça

Você se refere à descoberta da cura de todos os males, não? Penso que sim, bem como a erradicação total da fome, da miséria e da ignorância. Mas ainda demoram vários séculos de progresso civilizatório.

ACREDITA QUE A TERRA PODERÁ COLIDIR COM OUTRO PLANETA OU CORPO CELESTE‎ Paulo Ricardo Fogaça

Poderá sim, mas isso é bem improvável. Com outro planeta é quase impossível. Mas com algum asteroide seria possível, como já houve. Isso poderia ser bem catastrófico, provocando grandes mudanças climáticas ou, conforme o tamanho, até alterações oceânicas e continentais. Todavia a probabilidade de uma colisão dessa monta é ínfima.

O QUE O SR ACHA SOBRE A EXISTENCIA DE ATLÂNTIDA, SERIA REALMENTE UMA CIVILIZAÇÃO MODERNA E AVANÇADA NA ÉPOCA OU SOMENTE MITO E FOLCLORE‎ Paulo Ricardo Fogaça

Mito e folclore.
Uma sugestão: use texto em caixa baixa (minúsculas). Fica melhor.

Professor, na sua opinião, se a biblioteca de Alexandria não fosse destruída, todos os campos da ciência hoje em dia estaria expressivamente mais avançado?‎ Fernando

Penso que sim. Especialmente se a Cristandade de então fosse tolerante com relação ao paganismo e às demais religiões, bem como à Filosofia e à Ciência livres de dogmas de qualquer religião. O que entravou, e que levou à destruição da Biblioteca de Alexandria, foi a transforação do Cristianismo de uma seita perseguida em uma religião perseguidora. Aí é que ele se corrompeu e deixou de ser o mensageiro de uma boa nova. Passou a se comportar como se comportava antes a religião oficial do Império Romano. Prostituiu-se. Engessou-se. Antolhou-se. Isso foi uma lástima para a humanidade. Depois veio o Islã e passou a fazer o mesmo. Apesar de, no início, o Islã até ser mais tolerante do que o Cristianismo.

Lendo uma pergunta que lhe fizeram sobre a pessoa nascer ou não má, lembrei de um vídeo de um rapper chamado Eduardo Taddeo em que ele diz que não vê "favelado" com genética propensa ao banditismo, ele vê uma situação que os transforma em bandidos. O que acha a respeito disso?‎ Pimenta, Felipe

Verdade, em parte. Nem todo favelado é bandido. Aliás, a minoria. E todos estão submetidos ao mesmo ambiente. Há quem tenha propensão para o mal sim. Mas não significa que vá ser mau. Pode resistir. E há quem tenha propensão para o bem e acabe sendo mau por influência do ambiente. Não se pode simplificar o assunto. Tanto há o fator genético quanto a situação. E eles não são determinantes. Apenas condicionantes ou, até, estimulantes. Mas a escolha ainda tem um certo grau de liberdade que não tira de todo a culpa por ela. Mas a atenua. Um dos grandes problemas de certos sociólogos é uma espécie de simplificação das explicações de fenômenos bem complexos. Isso também acontece com psicólogos e economistas. A realidade psicológica, sociológica e econômica, bem como política e histórica, é extremamente complexa. A formulação de modelos explicativos simplista é uma grande temeridade.

http://ask.fm/wolfedler/answer/51880476445 Nunca sentiu ciúme de alguém ? Não acha que seria um ato voluntário do sentimento do amor?‎ Victória Miotto

De modo nenhum. Nunca senti ciúme. Ciúme não provém do amor mas do egoísmo. Do fato de se querer que o ser amado nos ame com exclusividade. É uma manifestação do desejo de posse do ser amado. Quem ama de verdade, quer o ser amado totalmente livre para amar em retribuição ou não e, mesmo amando em retribuição, para amar, também a outras pessoas. Refiro-me ao amor erótico-romântico e não ao fraternal, filial, paternal ou maternal, pois esses se admite que sejam plurais. Mas o romântico também pode. Quantas pessoas não sentem grande infelicidade por terem que optar entre dois amores igualmente amados em função dessa estúpida convenção de que amor tem que ser único. As pessoas, naturalmente, podem amar a mais de uma outra com toda a sinceridade, intensidade e abrangência. Não há razão para se impedir isso. Então não se pode exigir de ninguém que, amando-nos, ame exclusivamente a nós. Quem tem ciúme não é porque ama, mas porque quer a posse exclusiva do amado. E isso significa que, de fato, não ama. O que não se pode é dizer que se ama exclusivamente a uma pessoa quando se ama também a outra. Tudo tem que ser aberto, conhecido e consentido. Sem o menor problema. Sem o menor ciúme. Deixe seu namorado ter outras namoradas, se ele ama e tenha você também outros namorados, se você os ama a todos. Mas nada escondido. Não estou dizendo que deva ser assim, mas que pode ser assim.

Conselho aos jovens para ter exito nas conquistas profissionais?‎ Thiago Monteiro Pereira

Jamais se dedicar a uma profissão de que não goste só porque dá mais dinheiro. Faça o que, de fato, seja aquilo que você até pagaria para fazer e, então você vai ganhar para fazer e fazer muito bem feito. Com isso alcançará a prosperidade e, muito mais importante, a felicidade, por fazer de sua profissão a sua própria vida e não algo separado, como muitos acham, erroneamente, que deva ser. A escolha da profissão tem que ser emocional e não racional. Senão vai se ter uma vida infeliz e ela é única. Não compensa de modo nenhum. E, feita a escolha errada, é mais difícil mudar de profissão do que de cônjuge.

Deveria haver matérias de humanas nos cursos de exatas?‎ Anônimo

Sim, como filosofia e administração. Português eu acho que não seria preciso, porque é obrigação sair da Educação Básica sabendo-a muito bem. Mas tem que ser exigido nos trabalhos a correção linguística, além do conteúdo específico. Disso eu não abro mão, isto é, de cortar ponto em prova de física e matemática por erro de português. Do mesmo modo que acho que conhecimentos de ciências exatas e biológicas do nível médio têm que ser considerados como sabidos nos cursos da área de humanas, já que o candidato passou no vestibular. A filosofia não é tanto para se conhecer sua história, mas para saber raciocinar e argumentar.

As pessoas tem culpa sobre por o que ela são? Por exemplo: Uma pessoa que é mal, ela tem culpa por ela ser mal? Ou ela nasceu assim? Se ela nasceu assim, ela não tem culpa, concorda?‎ Husky Siberiano

sso não é tão cristalino assim. A pessoa não nasce nem má e nem boa, mas sim com maior propensão a ser má ou ser boa. Mas ela pode não se tornar aquilo para o que seja propensa. Vai depender de sua interação com o resto do mundo e de sua história de vida. Em parte isso não está sob o seu controle e, em parte, está sim. Ela faz escolhas e essas escolhas são muito influenciadas, mas sempre há um fator livre e voluntário que pode aceitar ou rejeitar as tendências e as influências. Claro que o grau de culpabilidade precisa ser ponderado. Então a culpa por ser mau (e não mal) é menor em quem tenha tal propensão e cujo ambiente a favoreça. Por outro lado, o mérito de ser bom não é tão grande para quem já nasceu com essa propensão e viveu num ambiente que a estimula. De qualquer modo, a sociedade não pode permitir, para sua salvaguarda, que pessoas se dediquem livremente a fazer o mal e a prejudicar os outros impunemente. É preciso que a sociedade crie um ambiente no qual a prática do mal seja severamente desvantajosa e a do bem seja proveitosa para seu praticante. Assim as tendências natas serão devidamente modificadas para estimular a prática do bem e a erradicação do mal. Mesmo sem culpa, ações malévolas precisam ser coibidas e uma das formas de se fazer isso é por meio de punições. Trata-se, simplesmente, de um comportamento pavloviano. Punir o mal comportamento e premiar o bom comportamento ainda é a maneira correta de se educar. Mas há duas ressalvas. A primeira e a definição isenta de bom e mau comportamento, que tem que ser feita de forma filosoficamente ética e não por conveniências pessoais dos cuidadores e nem da sociedade. Segundo é a dosagem adequada de pena e prêmio, para não provocar efeitos danosos de recalque e retaliação, bem como de vaidade e soberba.

É fisicamente impossível encostarmos em algo?‎ Pimenta, Felipe

A questão é definir o que significa "encostar". Quando dois sólidos se aproximam a uma distância da ordem de algumas centenas de átomos, por mais liso que eles sejam, exceto se forem planos de clivagem de cristais, as irregularidades se manifestam e alguns ressaltos penetram em algumas reentrâncias das suas superfícies. Nos pontos mais próximos, os átomos de cada um conseguem ficar a cerca de um átomo de distância dos átomos dos outros. Nessa distância, a repulsão entre os elétrons externos dos átomos de um e de outro sobrepuja a atração dos prótons de um pelos elétrons do outro e a resultante é repulsiva. Isso impede o toque dos átomos, exceto se houver combinação química, caso em que os dois sólidos se fundem em um só. Mas se eles se mantiverem distintos, de fato, os átomos não se tocam. Nem quando uma lâmina super afiada corta um pedaço de carne. Uma cola pode ser de dois tipos. No primeiro trata-se de um líquido que penetra nas irregularidades e se solidifica, "enganchando" um sólido no outro através da cola. No segundo ela reage quimicamente com ambos, promovendo uma continuidade de um para o outro. É o caso das cianoacrílicas. Assim, quando dizemos que dois sólidos se tocaram, eles estão a alguns átomos de distância um do outro. Mesmo assim um pode empurrar o outro, pela repulsão dos elétrons externos dos dois.

Como se livrar dos pensamentos negativos?‎ Husky Siberiano

Ocupando a mente com outros pensamentos. Especialmente fazendo alguma coisa. Quanto mais se esforçar para que eles não apareçam ou para se esquecer de algo, mais esse assunto fica vivo. Não há como provocar um esquecimento. Ele acontece espontaneamente por falta de evocação. Mas nunca totalmente. De qualquer modo, entretendo-se com outros assuntos, o que for indesejável não vem à tona.

Pesquisei no google alguma resposta sua sobre efeito placebo e não encontrei nada...‎ Gabriel Fernandes

Efeito placebo é uma ação psicológica sobre o organismo, capaz de provocar o incremento da ação do sistema imunológico de forma a proceder à melhoria de um estado patológico, pelo fato da pessoa acreditar que esteja sendo submetida a alguma terapia ou medicação sem que, de fato, esteja. Isso pode ser por meio de uma substância inócua administrada como se fora remédio ou, por exemplo, pela crença de que orações sejam capazes de curar, ou, mesmo, "cirurgias espirituais". O resultado, contudo, se limita a curas que o próprio organismo seja capaz de proceder.

Por que as pessoas acreditam que fazer pedidos para estrelas cadentes se realizam?‎ mih campos

Ora. Isso é só uma superstição sem nenhum significado que se pode fazer por brincadeira. Não conheço a origem dessa superstição. No filme "Pinnochio" de Walt Disney, tem uma música maravilhosa sobre essa crença. É de emocionar: "When you wish upon a star" (tem que entender inglês):

melhor maneira de cometer o suicido de que forma ? muitas pessoas dizem arma mas... ninguém conseguiria uma arma e nem veneno -' ajuda ?‎ Guilherme Amâncio

Não ajudo não. Acho que esta vida é uma preciosidade inigualável para se dar cabo a ela. Ela é única para cada pessoa. A morte é o fim total de tudo. O fato de existirmos e, ainda por cima, sermos humanos, com inteligência, criatividade, consciência, memória, vivendo e sociedade, tudo isso é um privilégio imenso e único que se tem que cuidar com o máximo desvelo. Por mais que se esteja infeliz, é preciso ver as contingências atuais de uma perspectiva afastada, para perceber que há muitas outras possibilidades e que tudo pode ser mudado. Então há que se ter ânimo para viver e fazer da vida uma alavanca para a melhoria do mundo, dando a ela um significado ímpar que só cada um pode fazer por si.

Poderia me explicar a diferença de impulso e energia?‎ Heitor Rocha

mpulso é uma das grandezas que mede a interação mecânica entre sistemas físicos, sendo definido pelo produto da força sofrida pelo tempo em que atua, se preciso, integrado ao longo do tempo. Trata-se de uma grandeza extensiva, que mede uma quantidade e não uma intensidade, como a força. Não é uma propriedade possuída por um sistema e sim por uma interação. O impulso total, de todas as forças atuantes em um sistema, vale o valor da variação de sua quantidade de movimento (momento linear) total. Se o sistema não interagir com nada, sua quantidade de movimento permanece constante no tempo. Já energia é um atributo, também extensivo, possuído por sistemas e não por interações. A ele se associa, também, uma grandeza, que mede a capacidade do sistema de realizar algo por meio de interações com outros, sejam mecânicas, térmicas ou de qualquer tipo. O valor da energia de um sistema pode variar quando ele interage com outros, sendo essa variação medida pelo trabalho mecânico que ele sofre ou pelo calor que ele receba de outros sistemas. Um conjunto de sistemas em interação também possui energia associada às interações experimentadas. Impulso, como trabalho e calor são medidas do que se transfere entre sistemas enquanto energia, quantidade de movimento ou momento angular são medidas do que sistemas ou conjuntos de sistemas possuem. Nenhum sistema possui trabalho, impulso ou calor. Veja isto: http://www.ruckert.pro.br/blog/?p=2045.

oque o senhor pensa sobre os gays?‎ Felix Khoury

Que se trata de um comportamento completamente normal, como o hetero, o bi e o assexual, quer seja por razão de uma orientação orgânica inata ou por uma opção consciente, mesmo em desacordo com a orientação. Apenas não aprecio os trejeitos histriônicos de alguns deles. O fato de desejar e se comprazer no amor erótico por alguém do mesmo sexo, para mim, não tem o menor problema, tanto quanto por ambos os sexos ou por nenhum. E se se trata de uma manifestação de amor, é algo muito melhor do que o preconceito contra tal comportamento que é uma manifestação de desamor.

Poderia me explicar o que seria Anarquismo e suas vertentes?‎ Heitor Rocha

Este artigo de dá, com toda a propriedade, um apanhado completo sobre o tema. Se quiser se aprofundar, consulte as ligações externas mencionadas no final do artigo:
http://pt.wikipedia.org/wiki/Anarquismo
Recomendo, também, as obras:
O que é Anarquismo - Caio Túlio Costa - Brasiliense (uma introdução)
História das Idéias e Movimentos Anarquistas - George Woodcock (2 vols.) - L&PM Pocket

a verdade e que quando tudo surgiu foi feito com escritas certas, em linhas tortas e que que ninguem sabe de nada‎ Waldo Damha Junior

Não. Quando tudo surgiu, surgiu sem ter sido feito por escrita nenhuma. Realmente ainda não se sabe o modo como se processou esse surgimento. O que não significa que não se possa vir a saber e nem que essa ignorância atual implique que se precise admitir que tenha surgido como obra de algum agente extrínseco ao Universo.

Tenha uma bela tarde, Ernesto. Uma pequena duvida em tercinas as três notas não precisam ter necessariamente a mesma duração?‎ Luisa

No meu entendimento, sim, têm que ter a mesma duração. Só que ela não é uma divisão do tempo em uma potência de 2, como 2, 4, 8, 16, 32, 64, como acontece com as figuras, em que cada qual tem a metade da duração da anterior. Em vez disso se usam outras frações, como 1/3, 1/5, 1/7 e assim por diante. Trata-se de uma execução mentalmente mais difícil de ser comandada pelo cérebro. Tente tamborilar com suas duas mãos. A esquerda batendo 4 vezes por segundo e a direita batendo 5 vezes por segundo. Não é fácil.

Foi encontrada à Arca de Noé. Você acha que é mais um item para se ter a crença da existência do Deus todo poderoso?‎ Joel Moreira.

Claro que não. Isso apenas confirma que a história narrada na Bíblia possua uma fundamentação fática, mas não que os detalhes da história sejam verdadeiros, pois isso não se verificou. Não se verificou que tal arca fora construída por ordem de deus. Não se verificou que tal arca abrigou um casal de todas as espécies animais terrestres do mundo inteiro. Não se verificou que tal arca foi projetada para resistir a um dilúvio que submergiu as terras do mundo inteiro. Nada disso fica confirmado. Muito menos a existência de Deus.

rs` porque passar em medicina, logo de primeira, é algo muito complicado!‎ Raelson Hipolito

Se você não se garante para passar em medicina pela primeira vez, acho melhor ficar um ano se preparando e entrar. Tem que meter meter a cara pra valer, sem refresco. Estudar, pelo menos, 10 horas por dia, incluindo sábados, domingos e feriados. Esquece o resto da vida. Aí você passa. Mas não se estresse. Vá firme e decidido com pertinácia e sem preguiça. Mas sem ansiedade.

O que o senhor acha daqueles ''ateus'' que acreditam na existência do diabo?‎ Falo tudo

Completamente incoerentes. Não há diabo sem deus.

Como é possível acreditar em um amor à distância? Será que realmente existe?‎ Biblioteca

Claro que sim. Isso não é uma questão de acreditar e sim de vivenciar. Muitos o vivenciam em toda a sua plenitude, sinceridade, amplitude, profundidade e completeza. Sob o aspecto sentimental, intelectivo e volitivo é tão verdadeiro quanto um amor vivenciado de modo presencial. Como antigamente acontecia por meio de cartas. A pessoa constrói uma imagem da outra em razão do que lhe é passado por ela e do que é percebido pelas interações à distância e dedica a essa imagem o amor como se, de fato, dedicaria à própria pessoa. Para quem ama e é amado, "é" a própria pessoa. Aliás, mesmo quem ama presencialmente, de fato ama uma imagem que faz da pessoa amada. A única diferença é a ausência do contato físico. Trata-se de um amor do tipo do "amor cortês" ou "cavalheiresco" dos cavaleiros medievais por sua dama. Um amor platônico. Mas tão intenso quanto um amor carnal, sem o contato.

observei na sua "bio" que diz ser "ateu", podes me explicar com suas sinceras palavras a origem da palavra "ateu" e qual sua visão sobre o assunto, eu sou ateu porquê não creio "na existência"de "theos". espero que consiga entender onde eu quis chegar!‎ damiam

Exatamente isso: Um ateu é quem não acredita que exista nenhuma espécie de divindade. Porque se a existência de alguma divindade fosse uma evidência, não se teria que crer ou não. Saber-se-ia que existe e não haveriam crentes e nem descrentes. Como não há crentes e nem descrentes da existência do Sol. Mas os deuses não se mostram. Então só se pode admiti-los por crença. Ateus são os que não possuem essa crença. É o meu caso.

Com tantas perguntas que o senhor recebe, muitas são com erro de português? Não sente vontade de corrigi-las?‎ Natasha Mattos

Realmente a fração de perguntas com erros de português é muito grande, mas eu não me fixo nesse aspecto e sim no conteúdo. Todavia, estou salvando todas para publicar, como comecei a fazer com as do formspring e, então, faço as devidas correções. Acho triste ver o descaso da juventude para com a língua portuguesa, pois penso que, ao sair do nível fundamental, todo mundo já deveria estar capacitado a escrever corretamente em nosso idioma. Nem precisava mais ter aulas de português no nível médio, pois nele não se aprende nada de novo em relação ao que se viu no fundamental. Bastaria que os professores das outras matérias cobrassem redações de seus conteúdos e, nelas, também corrigissem o português. Aliás sempre fiz isso em minhas provas de física. Eu pedia redações de física. Mas isso na faculdade, para os alunos do Curso de Física. É inadmissível um professor de qualquer matéria que não saiba escrever em bom português.
Veja isto: http://pt.scribd.com/doc/76733718/Pergunte-me

ateu de merda, voce acha q agente veio dos macacos? presta atençao, como explica q qd estamos tristes, um musculo q é o coraçao sente aperto??‎ -' StéeH

Não apenas vimos de macacos, mas "somos" macacos, já que macaco é o nome popular dos primatas, ordem biológica a que pertencemos, como os demais "macacos". Mas nossos ancestrais não são os macacos atuais. Todos foram extintos. Todavia nós e os outros macacos temos algum ancestral em comum. O fato do coração sofrer constrição e taquicardia em resposta a emoções advém de que fatos emocionais provocam, por um comando nervoso, a secreção de adrenalina pelas supra-renais, comandada pela secreção de ACTH pela hipófise, que, por sua vez, é comandada pelo hipotálamo.
Além da ação hormonal, as emoções também provocam ações do sistema vago-simpático, que estimulam ou reprimem o funcionamento dos órgãos internos e a musculatura lisa.
http://pt.wikipedia.org/wiki/Adrenalina.

sexta-feira, 27 de dezembro de 2013

O quê é a anti-matéria?‎ Lucas Gomes

Um sistema composto de partículas do tipo das partículas da matéria, mas com suas cargas elétricas invertidas, isto é, prótons negativos e elétrons positivos. A anti-matéria em si mesmo é tão estável quanto a matéria, mas, em contato com a matéria se aniquila, transformando cada par de partícula material e sua correspondente anti-partícula em dois fótons cuja energia equivale às massas destruídas pela equação E=mc². Esses fótons correspondem aos raios gama, de altíssima penetração. No surgimento do Universo, formaram-se quantidades iguais das duas, que se aniquilavam e logo surgiam outras. Todavia, muitas que surgiam eram instáveis e decaiam radioativamente antes de se aniquilar. Como há uma pequena diferença no tempo de decaimento de uma partícula e sua anti-partícula, no final das contas, depois que, com a expansão do Universo, a separação entre elas não permitia mais a aniquilação antes do decaimento, ficou uma sobra de partículas em relação a anti-partículas de uma parte em um bilhão. O resto todo se transformou nos fótons da chamada "radiação de fundo" do Universo, que ainda hoje pode ser detectada por um aparelho de TV, quando não está sintonizando canal nenhum. O chiado e o chuvisco da tela são os fótons do Universo.

Quando e de que você deu aula na UFJF?‎ Husky Siberiano

De Física Geral e Experimental III e IV (Eletricidade, Eletromagnetismo e Ótica), para os cursos de Física, Matemática, Química, Engenharia Elétrica e Engenharia Civil, de 1974 a 1976.

Você tem as respostas que aqui dá na "ponta da lingua" ou sempre raciocina muito e reflete sobre antes?‎ André Moura

Varia. Umas eu realmente já tenho "na ponta da língua". Outras eu preciso refletir e ponderar para responder. Outras, ainda, requerem que eu consulte minha biblioteca ou algum site da internet. Estas, em geral, eu vou deixando para depois, por falta de tempo para pesquisar. Há as que eu não sei mesmo e teria que estudar desde o início. Estas eu respondo dizendo que não sei. Finalmente há as que são completamente idiotas e estapafúrdias. São as que eu, simplesmente, apago.

"Dois excessos: excluir a razão, admitir apenas a razão". Concorda?‎ forgotten

Certamente que sim. Esse é o segredo da justa e sensata medida. Temperar a razão com a intuição e a emoção. Todas têm o seu valor e são requeridas para os processos decisórios e para caminhar com sabedoria.

Caro E. V. Rückert, se Igreja Católica tivesse menos poder na Idade Média, você não acha que o avanço da ciência seria mais rápido e próspero?‎ Radioactive

Certamente que sim. Muito mais. Se o paganismo tivesse prevalecido no Império Romano, primeiro que o mundo possivelmente já teria abandonado a religião há tempos. Segundo que, possivelmente, o islamismo não teria surgido. Então, sem essas duas religiões, o espírito inquiridor greco-romano teria possibilitado um progresso científico e tecnológico bem maior até hoje. Sem comparação. Essas religiões, com seus livros sagrados com a pretensão de serem depositários de toda a verdade, atravancam extremamente o livre exame do comportamento da natureza e da sociedade, que é a base da ciência. Só os que se libertaram desse dogmatismo é que conseguiram produzir o progresso. Mas às custas de muita dificuldade por terem que nadar contra a correnteza.

Bom, muitos dos seres humanos, muitos mesmo, não conseguiriam viver sem a Bíblia, ia ser uma catástrofe creio eu, infelizmente muitos de nós não conseguem acreditar na ciência, precisam de um ser onipotente para viver.‎ Gabriel Alves

Conseguem sim. Os cristão representam apenas 31% da humanidade, mesmo que seja a religião com mais adeptos. Isso significa que 69% da humanidade vive sem a Bíblia.

Sr. Ernesto, meu plano para o futuro é, passar no enem para farmacia, e ai, me especializar em quimica, daí, faço um vestibular para Medicina (meta: medico endocrinologista), estou certo em seguir esse caminho? um bom plano?‎ Raelson Hipolito

Porque você não entra para medicina de uma vez?

infelizmente muitos cientistas ainda temem a si mesmo porque nao conhecem nada do nada e e biblico que muitos procuraram o motivo da existencia, mas e so nao temer a deus, nao sejamos ignorantes dos fatos, porque se nao fosse a biblia criada por homens o mundo era outro‎ Waldo Damha Junior

Certamente que, sem a Bíblia, o mundo seria outro. A questão é que penso que seria melhor.

se nao fosse a biblia as constituicoes e leis mundiais seriam outras, talvez nem computador existisse, nem nos dois estariamos que nem tontos discutindo sobre este assunto, na verdade eu e voce tememos a deus eu creio e voce esta na duvida, procure deus e ele de consolara, jesus te ama‎ Waldo Damha Junior

Não sei em que a Bíblia teria alguma interferência na existência de qualquer tipo de tecnologia. Quanto às constituições e leis, há alguma interferência bíblica nas nações cristianizadas, mas ela é muito menor, por exemplo, do que a do Direito Romano, ou da Democracia Grega, que não têm nada a ver com a Bíblia. Sem contar que todo mundo oriental tem uma legislação completamente dissociada da Bíblia, baseada em Confúcio, no Tao Te Ching, nos Vedas, no Corão e outros livros.

deus e pura energia que ninguem sabe de onde vem tanta energia‎ Waldo Damha Junior

Não é não. Mesmo que existisse Deus, não seria pura energia por várias razões.
Para começar energia é um atributo físico de sistemas físicos e Deus não seria uma entidade física. Logo Deus não poderia possuir energia.
Segundo, energia não é algo que exista em si mesmo. Não é entitativa. É uma qualidade, um atributo, uma propriedade. Que certos sistemas podem ou não possuir. Não existe "energia pura", mas apenas energia que algo possua.
Terceiro, se houvesse Deus ele teria que ter alguma estrutura e algum funcionamento para pensar, poder fazer escolhas, para tomar decisões, para agir e atuar sobre o Universo, não se sabe de que modo. Uma entidade simples, constituída puramente de uma única substância, fosse o que fosse, não seria capaz de nada. Ocorrências se dão em razão de diferenciações estruturais que ensejam possíveis rearranjos que são em que se constituem as ocorrências e os fenômenos (classes de ocorrências). Se Deus é uma entidade que age, ele experimenta ocorrências internas, o que significa que não é uniforme, nem simples, nem puro.

sr Ernesto, mesmo tendo tantas provas, imagens, relatos de pessoas, medicos profissionais que revelaram que o governo dos Estados Unidos o forçaram a mentir, o Senhor nao acredita na existência de outros seres inteligentes? não acredita que existe muita coisa secreta relacionada a isso no nosso mun‎ Raelson Hipolito

Acho que existem fatos secretos sim. Mas não que se refiram a extra-terrestres. Se houvesse, os casos não seriam sempre nebulosos e pendentes de interpretações facilmente contestáveis. Seriam de evidência cristalina, totalmente indubitáveis. Não há nenhum assim.

A vida tem mesmo um sentido? Qual o fundamento de existir? Tudo acaba depois que morremos? Tudo é apenas biológico? (Pergunta do Facebook)‎ Ernesto von Rücker

Pelo que depreendo não há sentido extrínseco para a vida, exceto viver. Nós, humanos somos como todos os animais, exceto que temos uma mente bem mais desenvolvida. Mas isso é uma diferença quantitativa e não qualitativa. Não somos ontologicamente nada alem de animais. Portanto não temos uma natureza diferente, como uma alma imortal. De fato, ao morrermos, acaba tudo. Todas as nossas lembranças. Sim, tudo é apenas biológico. Todavia podemos deixar nosso legado para o mundo, em termos de nossas obras. E podemos atribuir um significado pessoal a nossas vidas, mesmo que ela não tenha um externo. Para mim esse significado reside no fato de sentirmos que o mundo se torna melhor pelo fato de existirmos. Isso preenche a nossa vida de sentido e apazígua nossa consciência em relação a esta sensação de vazio existencial.

A MORAL É RELATIVA? O CERTO E O ERRADO DEPENDEM DA ÉPOCA E DA CULTURA? (Pergunta do Facebook)‎ Ernesto von Rückert

A moral é relativa. Só que a moral não concerne ao certo e ao errado. Isso é com a ética. A moral cuida do que seja permitido e do que seja proibido. Deveria, mas nem sempre coincide com o certo e o errado. A moral varia com a época, o lugar e o estrato social. Já a ética, sendo filosófica, pretende nortear a moral de uma forma universal. Mas há preceitos morais, dependendo das circunstâncias, que não são éticos, bem como ações tidas como imorais que não ferem a ética. Uma ação é tida como ética se promove a maximização da felicidade para o maior número de seres, se pode ser erigida como preceito universal e se seja tal que queiramos ser dela objeto. Em geral um bem seria o que causasse prazer, alegria, lucro, benefício, vantagem, felicidade e um mal o que causasse tristeza, dor, prejuízo, malefício, desvantagem, infelicidade. Seria ético, contudo, uma ação que causasse um mal, se fosse em prol de um bem maior.

O que é a razão (pergunta que me fizeram no Facebook)?‎ Ernesto von Rückert

Razão é a faculdade mental de encaminhar o pensamento de modo a produzir conclusões a partir de certas suposições de uma forma comprovada e justificada. A razão não considera os afetos envolvidos, mas tão somente os motivos determinantes de forma lógica demonstrável. Isto é, não admite intuições e palpites injustificáveis, mesmo que levem a resultados corretos. O que não significa que a racionalidade seja o único critério válido de aceitação de uma decisão, que também tem que pesar, além do raciocínio, a emoção e a intuição.

Você acredita em algum tipo de fim para a humanidade? Digamos, a extinção das raças, onde o universo voltara a ser como antes.‎ Gabriel Alves

Novamente não é uma questão de acreditar e sim de se fazer projeções, com base nos conhecimentos existentes. Em média, as espécies vivas da Terra têm perdurado por 15 milhões da anos. Colocando a humanidade nessa média, como temos só 200 mil anos de idade, ainda nos restam 14 milhões e 800 mil anos de existência. Aí seremos extintos. Enquanto isso, certamente, outras espécies trans-humanas (evoluídas da nossa) ou mesmo de outros primatas, poderão surgir e haver a coexistência de mais de uma espécie inteligente. Essas nos substituirão e depois, outras as substituirão e assim por diante. Até que as condições ambientais do planeta não suportem mais vida nele. Isso acontecerá dentro de uns 3 ou 4 bilhões de anos, quando, após ter consumido todo o hidrogênio, o Sol passará a inchar e se transformar em uma estrela gigante vermelha, que irá crescendo e aquecendo a Terra cada vez mais até evaporar toda a água existente, inviabilizando a vida. Em certo momento a própria Terra será alcançada pela superfície do Sol e poderá se evaporar, deixando de existir. Ou então, se o Sol não crescer tanto, depois que ele fundir a série de elementos até o ferro, explodirá. A onda de choque, então, triturará a Terra, transformando-a em pedregulhos soltos no espaço. Mas o Universo, enquanto isso, não terá mudado muito. Nosso Sol terá dado apenas uma 20 voltas em torno da galáxia. Só que, dentro de 3 bilhões de anos, a galáxia de Andrômeda, em rota de colisão conosco, terá atingido a Via Láctea. Uma passará por dentro da outra, mas as colisões entre estrelas será rara, dado o seu afastamento.
Só que isso mudará totalmente a estrutura das duas galáxias.

Acredita no principio da reciprocidade, ou seja, tudo o que você faz volta para você de alguma forma?‎ Numinoso

Claro que não. Não há nada que indique que deva ser assim e nem a observação dos acontecimentos confirma tal assertiva. Muita gente faz um monte de maldades e se dá muito bem e muita gente faz um monte de bondades e se dá muito mal. Certamente que uma boa ação propicia condições de ser retribuída, do mesmo modo que uma má ação não favorece uma boa retribuição. Mas isso não é categórico para que possa ser chamado de "princípio".

Acreditas em exorcismo ? o que achas dessa prática?‎ Çabryna

Completamente inócua no que diz respeito a possessão, pois isso não existe. O que existe é encenação e esquizofrenia.

ACERTO

Ernesto, você acha que os jovens hoje em dia, estão se tornando alienados?‎ Stefani

Acho que a proporção dos alienados realmente está aumentando. Vejo cada vez mais jovens não se importando com nada a não ser com sua própria vidinha. Não ligam para a política, para os problemas ambientais e sociais, não se importam com sua própria vida futura, como se fossem viver a vida toda às custas dos pais. Isso é preocupante. Há exceções, é claro. Mas os que são conscientes são ridicularizados pela maioria, de modo que fica difícil manter uma atitude de personalidade diante do grupo. Um fato que me preocupa é o desinteresse pelo conhecimento. Vejo um futuro sombrio para o Brasil, quando o resto da atual geração que está com as rédeas do pais na mão vier a falecer, pois quem está vindo aí para substituir, no geral, está muito mal preparado e poderá levar nosso país a ficar completamente nas mãos das nações mais desenvolvidas, onde a capacidade seja um valor cultivado. Ou então o grosso da população ficar à mercê de uma minoria inteligente e bem instruída, mas não necessariamente bem intencionada. Um dos maiores esforços do processo educativo, para mim, é, justamente, conscientizar a juventude e tirá-la da alienação. Para o bem do futura dela mesma.

Você acha que religião, tem sua parte nas guerras?‎ Gabriel Alves

Muita. Religião e economia são os maiores fatores desencadeadores de guerras. A cobiça, o desejo de dominar e possuir a riqueza alheia. E a concepção de que quem não tenha a mesma fé deva ser exterminado para a glória de sua divindade. Que mentalidade mais tacanha! Mas eu ainda penso que, por meio da educação, as religiões serão abolidas e a prosperidade será compartilhada por todos, de modo que não haverão mais guerras.

Como explicar essa nova Epidemia chamada ''Bissexualismo''? Por que se expande tão rápido?‎ Radioactive

Nem estou sabendo disso. Para mim o comportamento sexual é inteiramente livre. Cada um seja como quiser. Ou por sua orientação inata ou por sua escolha consciente. Não tem problema nenhum. Só não pode ser forçado. E nem se justifica um comportamento apenas por modismo, para ir na onda de todo mundo. Quem age assim não tem personalidade. É uma pessoa boçal.

Bom levando em consideração a sua última resposta sobre as armas, e as armas biológicas(Não tenho certeza sobre o nome), quais serão as mais "úteis", digamos assim, em alguma próxima guerra.‎ Gabriel Alves

Para mim a próxima guerra deveria ser resolvida com luvas de boxe na mão dos líderes em conflito. Brincadeira à parte, para começar, a humanidade deverá se esforçar para não haver mais nenhuma guerra. Havendo alguma, para mim seria preciso, simplesmente, usar algum drone que atacasse diretamente o quartel general do líder belicoso e acabasse com a guerra sem ser preciso matar quase ninguém. Qualquer meio provocador de grande mortandade populacional tem que ser evitado a todo custo. Para mim, os soldados deveriam se recusar, na totalidade, a lutar. Assim não haveria guerra nenhuma. Guerra é um disparate que não tem justificativa nenhuma, exceto em defesa de um agressor. Mas agredir é algo inteiramente despropositado. Nenhuma população poderia concordar com qualquer governo agressor e impedi-lo de assim agir.

ACREDITA EM DESARMAMENTO MUNDIAL‎ Paulo Ricardo Fogaça

Sim, mas não para um futuro próximo e nem a médio prazo. Entendo, porém, que a humanidade concluirá, em um ou dois séculos, que não poderá mais viver em estado de tensão bélica e animosidade sob pena de se auto-destruir. Então todos se tolerarão mutuamente em suas diferentes concepções. Isso dependerá, crucialmente, de uma conclusão, por parte dos países ricos, que o mundo é um só e que é intolerável a coexistência da riqueza com a pobreza. Então passarão ao esforço para acabar com a pobreza. A prosperidade generalizada, bem como a elevação do nível educacional, fatalmente conduzirá à redução do fanatismo e ao estabelecimento da tolerância que trará a paz mundial.

LinkWithin

Related Posts with Thumbnails